Diabetes Mellitus Flashcards

1
Q

T2DM - Mx - Updated NICE Guidelines

A

NICE updated its guidance on the management of type 2 diabetes mellitus (T2DM) in 2015. Key points are listed below:

  • HbA1c targets have changed. They are now dependent on what antidiabetic drugs a patient is receiving and other factors such as frailty
  • There is more flexibility in the second stage of treating patients (i.e. after metformin has been started) - you now have a choice of 4 oral anti diabetic agents

NB: It’s worthwhile thinking of the average patient who is taking metformin for T2DM, you can titrate up metformin and encourage lifestyle changes to aim for a HbA1c of 48 mmol/mol (6.5%), but should only add a second drug if the HbA1c rises to 58 mmol/mol (7.5%)

How well did you know this?
1
Not at all
2
3
4
5
Perfectly
2
Q

T2DM - Dietary Advice

A

Dietary advice:

  • encourage high fibre, low glycaemic index sources of carbohydrates
  • include low-fat dairy products and oily fish
  • control the intake of foods containing saturated fats and trans fatty acids
  • limited substitution of sucrose-containing foods for other carbohydrates is allowable, but care should be taken to avoid excess energy intake
  • discourage use of foods marketed specifically at people with diabetes
  • initial target weight loss in an overweight person is 5-10%
How well did you know this?
1
Not at all
2
3
4
5
Perfectly
3
Q

T2DM Mx: HbA1c Targets

A

This is area which has changed in 2015
- individual targets should be agreed with patients to encourage motivation
- HbA1c should be checked every 3-6 months until stable, then 6 monthly
- NICE encourage us to consider relaxing targets on ‘a case-by-case basis, with particular consideration for people who are older or frail, for adults with type 2 diabetes’
- in 2015 the guidelines changed so HbA1c targets are now dependent on treatment:
LIFESTYLE OR DRUG TREATMENT

Lifestyle : HbA1c target 48 mmol/mol (6.5%)
Lifestyle + Metformin: HbA1c target 48 mmol/mol (6.5%)
Includes any drug which may cause hypoglycaemia (e.g. lifestyle + sulfonylurea): HbA1c target 53 mmol/mol (7.0%)
Already on one drug, but HbA1c has risen to 58 mmol/mol (7.5%): HbA1c target 53 mmol/mol (7.0%)

Practical examples:
A patient is newly diagnosed with HbA1c and wants to try lifestyle treatment first. You agree a target of 48 mmol/mol (6.5%)
You review a patient 6 months after starting metformin. His HbA1c is 51 mmol/mol (6.8%). You increase his metformin from 500mg bd to 500mg tds and reinforce lifestyle factors

How well did you know this?
1
Not at all
2
3
4
5
Perfectly
4
Q

T2DM - Drug Treatment

For pt who can tolerate Metformin:

A

Drug treatment

The 2015 NICE guidelines introduced some changes into the management of type 2 diabetes. There are essentially two pathways, one for patients who can tolerate metformin, and one for those who can’t.

Tolerates metformin:
metformin is still first-line and should be offered if the HbA1c rises to 48 mmol/mol (6.5%)* on lifestyle interventions
if the HbA1c has risen to 58 mmol/mol (7.5%) then a second drug should be added from the following list:
→ sulfonylurea
→ gliptin (DPP-4 Inhibitors)
→ pioglitazone
→ SGLT-2 inhibitor
if despite this the HbA1c rises to, or remains above 58 mmol/mol (7.5%) then triple therapy with one of the following combinations should be offered:
→ metformin + gliptin + sulfonylurea
→ metformin + pioglitazone + sulfonylurea
→ metformin + sulfonylurea + SGLT-2 inhibitor
→ metformin + pioglitazone + SGLT-2 inhibitor
→ OR insulin therapy should be considered

Criteria for glucagon-like peptide1 (GLP1) mimetic (e.g. exenatide)
if triple therapy is not effective, not tolerated or contraindicated then NICE advise that we consider combination therapy with metformin, a sulfonylurea and a glucagonlike peptide1 (GLP1) mimetic if:
→ BMI >= 35 kg/m² and specific psychological or other medical problems associated with obesity or
→ BMI < 35 kg/m² and for whom insulin therapy would have significant occupational implications or
weight loss would benefit other significant obesityrelated comorbidities
only continue if there is a reduction of at least 11 mmol/mol [1.0%] in HbA1c and a weight loss of at least 3% of initial body weight in 6 months

NB Insulin should not be combined with a GLP-1 mimetic.

Practical examples
you review an established type 2 diabetic on maximum dose metformin. Her HbA1c is 55 mmol/mol (7.2%). You do not add another drug as she has not reached the threshold of 58 mmol/mol (7.5%)
a type 2 diabetic is found to have a HbA1c of 62 mmol/mol (7.8%) at annual review. They are currently on maximum dose metformin. You elect to add a sulfonylurea

Example Names:

Sulfonylurea: Gliclazide, Tolbutamide, Glibenclamide, Glipizide, Glimerpiride

SGLT-2 Inhibitors: Dapagliflozin, Canaglifozin, Empagliflozin

Pioglitazone = Only thiazolidinedione licensed for DM

DPP-4 Inhibitors (Gliptins) = Sitagliptin, Linagliptin, Vildagliptin

GLP-1 Mimetic = Exenatide, Liraglutide, Lixisenatide, Dulaglutide

How well did you know this?
1
Not at all
2
3
4
5
Perfectly
5
Q

T2DM - Drug Treatment

For pt who cannot tolerate Metformin

A

The 2015 NICE guidelines introduced some changes into the management of type 2 diabetes. There are essentially two pathways, one for patients who can tolerate metformin, and one for those who can’t.

Cannot tolerate metformin or contraindicated (NB eGFR<30ml/minute/1.73m2 metformin is contraindicated)
If the HbA1c rises to 48 mmol/mol (6.5%)* on lifestyle interventions, consider one of the following:
→ sulfonylurea
→ gliptin
→ pioglitazone
if the HbA1c has risen to 58 mmol/mol (7.5%) then a one of the following combinations should be used:
→ gliptin + pioglitazone
→ gliptin + sulfonylurea
→ pioglitazone + sulfonylurea
if despite this the HbA1c rises to, or remains above 58 mmol/mol (7.5%) then consider insulin therapy

Starting insulin
metformin should be continued. In terms of other drugs NICE advice: ‘Review the continued need for other blood glucose lowering therapies’
NICE recommend starting with human NPH insulin (isophane, intermediate acting) taken at bed-time or twice daily according to need

*this is a bit confusing because isn’t the diagnostic criteria for T2DM HbA1c 48 mmol/mol (6.5%)? So shouldn’t all patients be offered metformin at diagnosis? Our interpretation of this is that some patients upon diagnosis will elect to try lifestyle measures, which may reduce their HbA1c below this level. If it then rises to the diagnostic threshold again metformin should be offered

How well did you know this?
1
Not at all
2
3
4
5
Perfectly
6
Q

T2DM - Risk Factor Modification

A

Risk factor modification

Blood pressure
target is < 140/80 mmHg (or < 130/80 mmHg if end-organ damage is present)
ACE inhibitors are first-line

Antiplatelets
should not be offered unless a patient has existing cardiovascular disease

Lipids
following the 2014 NICE lipid modification guidelines only patients with a 10-year cardiovascular risk > 10% (using QRISK2) should be offered a statin. The first-line statin of choice is atorvastatin 20mg on

How well did you know this?
1
Not at all
2
3
4
5
Perfectly
7
Q

Insulin Therapy

A

Insulin therapy

Insulin therapy revolutionised the management of diabetes mellitus when it was developed in the 1920’s. It is still the only available treatment for type 1 diabetes mellitus (T1DM) and is widely used in type 2 diabetes mellitus (T2DM) where oral hypoglycaemic agents fail to gain adequate control.

It can sometimes seem daunting to understand the various types of insulin but it is important you have a basic grasp to avoid potential harm to patients.

How well did you know this?
1
Not at all
2
3
4
5
Perfectly
8
Q

Classification of Insulin

A

Classification of insulin

By manufacturing process
porcine: extracted and purified from pig pancreas
human sequence insulin: either produced by enzyme modification of porcine insulin (emp) or biosynthetically by recombinant DNA using bacteria (crb, prb) or yeast (pyr)
analogues

By duration of action

Onset > Peak > Duration

Rapid-acting insulin analogues:

  • 5 mins
  • 1 hour
  • 3-5 hours

Short-acting insulin

  • 30 mins
  • 3 hours
  • 6-8 hours

Intermediate-acting insulin

  • 2 hours
  • 5-8 hours
  • 12-18 hours

Long-acting insulin analogues

  • 1-2 hours
  • Flat profile
  • Up to 24 hours

Premixed preparations - - -

Patients often require a mixture of preparations (e.g. both short and long acting) to ensure stable glycaemic control throughout the day.

How well did you know this?
1
Not at all
2
3
4
5
Perfectly
9
Q

Rapid-acting Insulin

A

Rapid-acting insulin analogues
the rapid-acting human insulin analogues act faster and have a shorter duration of action than soluble insulin (see below)
may be used as the bolus dose in ‘basal-bolus’ regimes (rapid/short-acting ‘bolus’ insulin before meals with intermediate/long-acting ‘basal’ insulin once or twice daily)
insulin aspart: NovoRapid
insulin lispro: Humalog

How well did you know this?
1
Not at all
2
3
4
5
Perfectly
10
Q

Short-acting Insulin

A

Short-acting insulins
soluble insulin examples: Actrapid (human, pyr), Humulin S (human, prb)
may be used as the bolus dose in ‘basal-bolus’ regimes

How well did you know this?
1
Not at all
2
3
4
5
Perfectly
11
Q

Intermediate-acting Insulin

A

Intermidate-acting insulins
isophane insulin
many patients use isophane insulin in a premixed formulation with

How well did you know this?
1
Not at all
2
3
4
5
Perfectly
12
Q

Long-acting Insulin

A
Long-acting insulins
insulin determir (Levemir): given once or twice daily
insulin glargine (Lantus): given once daily
How well did you know this?
1
Not at all
2
3
4
5
Perfectly
13
Q

Insulin - Pre-mixed Preparations

A

Premixed preparations
combine intermediate acting insulin with either a rapid-acting insulin analogue or soluble insulin
Novomix 30: 30% insulin aspart (rapid-acting), 70% insulin aspart protamine (intermediate-acting)
Humalog Mix25: 25% insulin lispro (rapid-acting), 75% insulin lispro protamine (intermediate-acting); Humalog Mix50: 50% insulin lispro, 50% insulin lispro protamine
Humulin M3: biphasic isophane insulin (human, prb) - 30% soluble (short-acting), 70% isophane (intermediate-acting)
Insuman Comb 15: biphasic isophane insulin 9human, prb) - 30% soluble (short-acting), 70% isophane (intermediate-acting)

How well did you know this?
1
Not at all
2
3
4
5
Perfectly
14
Q

Administration of Insulin

A

Administration of insulin

The vast majority of patients administer insulin subcutaneously. It is important to rotate injection sites to prevent lipodystrophy. Insulin pumps are available (‘continuous subcutaneous insulin infusions’) which delivers a continuous basal infusion and a patient-activated bolus dose at meal times.

Intravenous insulin is used for patients who are acutely unwell, for example with diabetic ketoacidosis. Inhaled insulin is available but not widely used and oral insulin analogues are in development but have considerable technical hurdles to clear.

How well did you know this?
1
Not at all
2
3
4
5
Perfectly
15
Q

Target Blood Sugars - Example Question

A

A 19-year-old man is reviewed prior to discharge. He presented with vomiting and abdominal pain and was found to have diabetic ketoacidosis. He was managed as an inpatient for five days before being well enough for discharge. He is also diagnosed with type 1 diabetes mellitus on this admission as a cause of the diabetic ketoacidosis. He has been educated by the diabetic nurse on how to manage his diabetes and insulin at home, but he is concerned about what his target plasma glucose should be after eating.

What is the recommended target after eating to be achieved by home monitoring?

	3-6mmol/litre
	5-10mmol/litre
	> 5-9mmol/litre
	7-12mmol/litre
	2-9 mmol/litre

The correct answer is 5-9mmol/litre. NICE recommends that people with type 1 diabetes should aim for 5-7mmol/litre on waking, 4-7mmol/litre before meals and 5-9mmol/litre 90 minutes after eating. Frequent testing is very important in patients starting with insulin therapy to avoid both high and low sugar levels.

Source:

‘Type 1 diabetes in adults: diagnosis and management’ Clinical guideline [NG17]. The National Institute for Health and Care Excellence, August 2015.

How well did you know this?
1
Not at all
2
3
4
5
Perfectly
16
Q

DM in Pregnancy

A

Pregnancy: diabetes mellitus

Diabetes mellitus may be a pre-existing problem or develop during pregnancy, gestational diabetes. It complicates around 1 in 40 pregnancies. NICE updated the guidance in 2015

Risk factors for gestational diabetes
BMI of > 30 kg/m²
previous macrosomic baby weighing 4.5 kg or above
previous gestational diabetes
first-degree relative with diabetes
family origin with a high prevalence of diabetes (South Asian, black Caribbean and Middle Eastern)

How well did you know this?
1
Not at all
2
3
4
5
Perfectly
17
Q

Gestational Diabetes - Screening and Diagnosis

A

Screening for gestational diabetes
women who’ve previously had gestational diabetes: oral glucose tolerance test (OGTT) should be performed as soon as possible after booking and at 24-28 weeks if the first test is normal. NICE also recommend that early self-monitoring of blood glucose is an alternative to the OGTTs
women with any of the other risk factors should be offered an OGTT at 24-28 weeks

Diagnostic thresholds for gestational diabetes
these have recently been updated by NICE, gestational diabetes is diagnosed if either:
fasting glucose is >= 5.6 mmol/l
2-hour glucose is >= 7.8 mmol/l

How well did you know this?
1
Not at all
2
3
4
5
Perfectly
18
Q

Mx of Gestational Diabetes and Pre-existing Diabetes in Pregnancy

A

Management of gestational diabetes:

  • Newly diagnosed women should be seen in a joint diabetes and antenatal clinic within a week
  • Women should be taught about selfmonitoring of blood glucose
  • Advice about diet (including eating foods with a low glycaemic index) and exercise should be given
  • If the fasting plasma glucose level is < 7 mmol//l a trial of diet and exercise should be offered
  • If glucose targets are not met within 1-2 weeks of altering diet/exercise metformin should be started
  • If glucose targets are still not met insulin should be added to diet/exercise/metformin
  • If at the time of diagnosis the fasting glucose level is >= 7 mmol/l insulin should be started
  • If the plasma glucose level is between 6-6.9 mmol/l, and there is evidence of complications such as macrosomia or hydramnios, insulin should be offered
  • Glibenclamide should only be offered for women who cannot tolerate metformin or those who fail to meet the glucose targets with metformin but decline insulin treatment

Management of pre-existing diabetes

  • Weight loss for women with BMI of > 27 kg/m^2
  • Stop oral hypoglycaemic agents, apart from metformin, and commence insulin
  • Folic acid 5 mg/day from pre-conception to 12 weeks gestation
  • Detailed anomaly scan at 20 weeks including four-chamber view of the heart and outflow tracts
  • Tight glycaemic control reduces complication rates
  • Treat retinopathy as can worsen during pregnancy
How well did you know this?
1
Not at all
2
3
4
5
Perfectly
19
Q

Targets for self monitoring of pregnant women (pre-existing and gestational diabetes)

A

Targets for self monitoring of pregnant women (pre-existing and gestational diabetes)

Time Target
Fasting 5.3 mmol/l
1 hour after meals 7.8 mmol/l, or:
2 hour after meals 6.4 mmol/l

How well did you know this?
1
Not at all
2
3
4
5
Perfectly
20
Q

Mx of Gestational Diabetes - Example Question

A

You review a 28-year-old woman who is 26 weeks pregnant. She has just had a routine oral glucose tolerance test as her BMI is 34 kg/m². The following results were obtained:

Time (hours) Blood glucose (mmol/l)
0 7.4
2 11.2

There have been no other antenatal problems and her anomaly scan was normal. What is the most appropriate action?

Repeat oral glucose tolerance test in 4 weeks
Start metformin + advice about diet / exercise + self-monitor glucose levels
Advice about diet / exercise + self-monitor glucose levels
> Start insulin + advice about diet / exercise + self-monitor glucose levels
Reassure results within normal limits

NICE have recently changed their gestational diabetes guidelines. Insulin should be started in the fasting glucose is >= 7 mmol/l. Aspirin should also be considered given the increased risk of pre-eclampsia.

How well did you know this?
1
Not at all
2
3
4
5
Perfectly
21
Q

Insulin Therapy - Example Question

A

A 34 year-old woman is seen by her GP for the annual review of her type 1 diabetes. Her most recent HbA1c is 58 mmol/mol and the only problem she has noticed are severe hypoglycaemic episodes during the night around 2-4am, that she noticed she was getting while working night shifts for a local superstore. However, by breakfast time, her blood glucose levels often rise to around 15 mmol/mol. Her insulin regimen is currently a twice daily mixed insulin.

What is the most appropriate change to her current insulin treatment?

	> Move to a basal bolus of insulin
	Take 1 dextrose tablet at 9pm
	Reduce nocturnal insulin dose
	Reduce morning insulin dose
	Add gliclazide

This patient is experiencing severe nocturnal hypoglycaemic episodes followed by rises in blood glucose by breakfast time. The most appropriate change to the insulin treatment regimen is changing to a basal bolus regime, with one daily injection of background long-acting insulin and three short acting injections of insulin.

How well did you know this?
1
Not at all
2
3
4
5
Perfectly
22
Q

Diabetes and the New Drugs

A

Diabetes mellitus: GLP-1 and the new drugs

A number of new drugs to treat diabetes mellitus have become available in recent years. Much research has focused around the role of glucagon-like peptide-1 (GLP-1), a hormone released by the small intestine in response to an oral glucose load

Whilst it is well known that insulin resistance and insufficient B-cell compensation occur other effects are also seen in type 2 diabetes mellitus (T2DM). In normal physiology an oral glucose load results in a greater release of insulin than if the same load is given intravenously - this known as the incretin effect. This effect is largely mediated by GLP-1 and is known to be decreased in T2DM.

Increasing GLP-1 levels, either by the administration of an analogue (glucagon-like peptide-1, GLP-1 mimetics, e.g. exenatide) or inhibiting its breakdown (dipeptidyl peptidase-4 ,DPP-4 inhibitors - the gliptins), is therefore the target of two recent classes of drug.

How well did you know this?
1
Not at all
2
3
4
5
Perfectly
23
Q

GLP-1 Mimetics eg Exenatide

A

Glucagon-like peptide-1 (GLP-1) mimetics (e.g. exenatide)

Exenatide is an example of a glucagon-like peptide-1 (GLP-1) mimetic. These drugs increase insulin secretion and inhibit glucagon secretion. One of the major advances of GLP-1 mimetics is that they typically result in weight loss, in contrast to many medications such as insulin, sulfonylureas and thiazolidinediones.

Exenatide must be given by subcutaneous injection within 60 minutes before the morning and evening meals. It should not be given after a meal.

Liraglutide is the other GLP-1 mimetic currently available. One the main advantages of liraglutide over exenatide is that it only needs to be given once a day.

Both exenatide and liraglutide may be combined with metformin and a sulfonylurea. Standard release exenatide is also licensed to be used with basal insulin alone or with metformin. Please see the BNF for a more complete list of licensed indications.

NICE state the following:

Consider adding exenatide to metformin and a sulfonylurea if:
BMI >= 35 kg/m² in people of European descent and there are problems associated with high weight, or
BMI < 35 kg/m² and insulin is unacceptable because of occupational implications or weight loss would benefit other comorbidities.

NICE like patients to have achieved a 11 mmol/mol (1%) reduction in HbA1c and 3% weight loss after 6 months to justify the ongoing prescription of GLP-1 mimetics.

The major adverse effect of GLP-1 mimetics is nausea and vomiting. The Medicines and Healthcare products Regulatory Agency has issued specific warnings on the use of exenatide, reporting that is has been linked to severe pancreatitis in some patients.

How well did you know this?
1
Not at all
2
3
4
5
Perfectly
24
Q

DPP-4 Inhibitors eg Vildagliptin

A

Dipeptidyl peptidase-4 (DPP-4) inhibitors (e.g. Vildagliptin, sitagliptin)

Key points
oral preparation
trials to date show that the drugs are relatively well tolerated with no increased incidence of hypoglycaemia
do not cause weight gain

NICE guidelines on DPP-4 inhibitors
NICE suggest that a DPP-4 inhibitor might be preferable to a thiazolidinedione if further weight gain would cause significant problems, a thiazolidinedione is contraindicated or the person has had a poor response to a thiazolidinedione

How well did you know this?
1
Not at all
2
3
4
5
Perfectly
25
Q

T2DM Mx - Example Question

A

A 60-year-old taxi driver presents to the diabetes clinic for review. He is obese with a body mass index of 36 kg/m², and has noticed a significant increase in his fasting blood glucose over the past 3 months from 6.5 mmol/l, to over 9 mmol/l. Apart from hypertension he has no other significant past medical history. Current medication includes metformin 1g BD, ramipril 10mg OD and atorvastatin 20mg OD. On examination his blood pressure is 155/82 mmHg, pulse is 70 beats per minute and regular. His chest is clear, abdomen is soft and non-tender with no masses, and he has no ankle swelling. Bloods reveal a normal creatinine and an HbA1c of 70 mmol/mol.

Which of the following is the most appropriate way to manage his blood glucose?

	Gliclazide
	> Liraglutide
	Sitagliptin
	BD mixed insulin
	Acarbose

This patient is obese with a significantly elevated HbA1c and has a sedentary lifestyle. Given his occupation as a taxi driver, he requires blood glucose lowering which won’t increase risk of hypoglycaemia, is potent, and preferably promotes weight loss. As such liraglutide, GLP-1 agonist is the most appropriate option, bringing long-acting insulin like glucose control, coupled with weight reduction of approximately 3% over 6 months.

Both BD mixed insulin and gliclazide increase the risk of hypoglycaemia and potentially lead to weight gain, and could impact on his driving licence, as such they should be avoided in this situation. Sitagliptin offers only a modest reduction in HbA1c, and acarbose results in flatulence in a significant proportion of users. As such these are better avoided as therapeutic options.

How well did you know this?
1
Not at all
2
3
4
5
Perfectly
26
Q

T2DM - Drug Treatment

A

Drug treatment

The 2015 NICE guidelines introduced some changes into the management of type 2 diabetes. There are essentially two pathways, one for patients who can tolerate metformin, and one for those who can’t:

Tolerates metformin:
metformin is still first-line and should be offered if the HbA1c rises to 48 mmol/mol (6.5%)* on lifestyle interventions
if the HbA1c has risen to 58 mmol/mol (7.5%) then a second drug should be added from the following list:
→ sulfonylurea
→ gliptin
→ pioglitazone
→ SGLT-2 inhibitor
if despite this the HbA1c rises to, or remains above 58 mmol/mol (7.5%) then triple therapy with one of the following combinations should be offered:
→ metformin + gliptin + sulfonylurea
→ metformin + pioglitazone + sulfonylurea
→ metformin + sulfonylurea + SGLT-2 inhibitor
→ metformin + pioglitazone + SGLT-2 inhibitor
→ OR insulin therapy should be considered

Criteria for glucagon-like peptide1 (GLP1) mimetic (e.g. exenatide)
if triple therapy is not effective, not tolerated or contraindicated then NICE advise that we consider combination therapy with metformin, a sulfonylurea and a glucagonlike peptide1 (GLP1) mimetic if:
→ BMI >= 35 kg/m² and specific psychological or other medical problems associated with obesity or
→ BMI < 35 kg/m² and for whom insulin therapy would have significant occupational implications or
weight loss would benefit other significant obesityrelated comorbidities
only continue if there is a reduction of at least 11 mmol/mol [1.0%] in HbA1c and a weight loss of at least 3% of initial body weight in 6 months

Practical examples
you review an established type 2 diabetic on maximum dose metformin. Her HbA1c is 55 mmol/mol (7.2%). You do not add another drug as she has not reached the threshold of 58 mmol/mol (7.5%)
a type 2 diabetic is found to have a HbA1c of 62 mmol/mol (7.8%) at annual review. They are currently on maximum dose metformin. You elect to add a sulfonylurea

Cannot tolerate metformin or contraindicated
if the HbA1c rises to 48 mmol/mol (6.5%)* on lifestyle interventions, consider one of the following:
→ sulfonylurea
→ gliptin
→ pioglitazone
if the HbA1c has risen to 58 mmol/mol (7.5%) then a one of the following combinations should be used:
→ gliptin + pioglitazone
→ gliptin + sulfonylurea
→ pioglitazone + sulfonylurea
if despite this the HbA1c rises to, or remains above 58 mmol/mol (7.5%) then consider insulin therapy

Starting insulin
metformin should be continued. In terms of other drugs NICE advice: ‘Review the continued need for other blood glucose lowering therapies’
NICE recommend starting with human NPH insulin (isophane, intermediate acting) taken at bed-time or twice daily according to need

How well did you know this?
1
Not at all
2
3
4
5
Perfectly
27
Q

T2DM Mx - Example Question

A

A 45-year-old man was commenced on 500mg metformin BD for type 2 diabetes mellitus as he was unable to meet his target HBA1c of 6.5% 6 months ago. He is now presenting for his regular follow up and feels well. He states that he has been compliant with his metformin, maintaining a healthy diet and exercising more regularly. His latest HBA1c is 7.3% and the rest of his investigations are normal. What is the next best step to manage his diabetes?

	> Increase his dose of metformin
	Add pioglitazone
	Make no change to his current treatment
	Add a sulfonylurea
	Advise to increase frequency of exercise

As per NICE guidelines on the management of type 2 diabetes in adults, at an HBA1c of 7.3% and a relatively low dose of metformin, the first step would be to increase his dose of metformin. His aim should still be an HBA1c of 6.5% on metformin.

Additional medications are only required if his HBA1c is 7.5% or higher, therefore the addition of pioglitazone or a sulfonylurea is incorrect. Whilst regular exercise is important, it is unlikely to be sufficient at this stage.

Click on the link below for more information and the algorithm for blood glucose lowering therapy in adults with type 2 diabetes:

https://www.nice.org.uk/guidance/ng28/chapter/1-Recommendations#drug-treatment-2

How well did you know this?
1
Not at all
2
3
4
5
Perfectly
28
Q

Testing for Gestational Diabetes - Example Question

A

A lady who is 10 weeks pregnant presents to her antenatal appointment asking for advice regarding gestational diabetes. She is a 31 year old English lady with a BMI (body mass index) of 28.7. In terms of family history she has a cousin who has type 1 diabetes mellitus and an aunt who is being treated for breast cancer. She has had two previous pregnancies, the first one she unfortunately miscarried at 8 weeks, and the second was a normal pregnancy that she took to term with a birth weight of 4.6kg. Neither of these pregnancies was complicated with gestational diabetes, and the baby is now 2 years old and has not had to be taken to see a doctor other than routine appointments.

What is the most appropriate testing regime for ruling out gestational diabetes in this woman?

> Oral glucose tolerance test at 24-28 weeks pregnant
None - as she has no risk factors for gestational diabetes
Oral glucose tolerance test at 12-14 weeks pregnant
Self-monitoring of sugars and repeat appointment in 2 weeks
HBa1c

This question requires knowledge on the risk factors for developing gestational diabetes and appropriate testing based on risk:

Risk factors for gestational diabetes include:
BMI >30kg/m²
Previous delivery of a baby over 4.5kg - which qualifies this patient
Previous gestational diabetes
Family history of diabetes (1st degree relative)
Minority ethnic family origin with a high prevalence of diabetes

If any one of these risk factors is present then one should offer testing for gestational diabetes. The gold standard testing for patients with risk factors is 2-hour 75g oral glucose tolerance test (OGTT) at 24-28 weeks gestation. If the patient has had gestational diabetes in a previous pregnancy then early-self monitoring of blood glucose or OGTT as soon as possible after booking could also be used for diagnosis.

A diagnosis of gestational diabetes is made if the patient has either:
Fasting glucose of 5.6 mmol/L or above OR
A 2-h plasma glucose of 7.8 mmol/L or above

How well did you know this?
1
Not at all
2
3
4
5
Perfectly
29
Q

Osteomyelitis

A

Osteomyelitis

Osteomyelitis describes an infection of the bone.

Staph. aureus is the most common cause except in patients with sickle-cell anaemia where Salmonella species predominate.

Predisposing conditions
diabetes mellitus
sickle cell anaemia
intravenous drug user
immunosuppression due to either medication or HIV
alcohol excess

Investigations
MRI is the imaging modality of choice, with a sensitivity of 90-100%

Management
flucloxacillin for 6 weeks
clindamycin if penicillin-allergic

How well did you know this?
1
Not at all
2
3
4
5
Perfectly
30
Q

Diabetic Cx: Osteomyelitis - Example Question

A

A 58-year-old man who has no fixed abode comes to the Emergency department because he is unable to walk. He has a history of alcoholism and type 2 diabetes. His main complaint is that his shoes have worn out and because of loss of sensation he didn’t notice that he had stepped on a nail. In total the lesion on his right foot has been present for approximately 3 weeks.

Which of the following is the next step in evaluating his foot injury?

	Inflammatory markers
	MRI foot
	> Plain x-ray foot
	USS foot
	Wound swab

The key next step here is to gather useful information about the extent of any foot infection. By 3 weeks post injury, changes consistent with osteomyelitis should be visible on plain x-ray. These may include soft tissue swelling, bone demineralisation, cortical irregularity, and an elevated periosteum.

Many patients progress from a plain x-ray on to MRI imaging of the foot for further evaluation of the extent of infection and to guide potential operative approaches for debridement. Inflammatory markers are a very non-specific marker of infection and ultrasound of the foot is only useful to visualise soft tissue swelling or collection of pus / fluid. A wound swab is likely to show a range of bacteria, this is what drives selection of a broader spectrum antibiotic such as co-amoxiclav in the diabetic population.

How well did you know this?
1
Not at all
2
3
4
5
Perfectly
31
Q

T2DM - Mx: Example Question

A

A 57-year-old man with a history of type 2 diabetes mellitus and chronic heart failure is reviewed in the diabetes clinic. His current medication list is as follows:

metformin 1g bd
gliclazide 160mg bd
ramipril 10mg od
bisoprolol 5mg od
furosemide 40mg od
simvastatin 20mg on

His annual bloods show the following:

Na+	140 mmol/l
K+	3.9 mmol/l
Urea	5.2 mmol/l
Creatinine	78 µmol/l
HbA1c	7.7% (61 mmol/mol)
Total cholesterol	4.2 mmol/l
HDL cholesterol	1.1 mmol/l

Blood pressure today is 124/78 mmHg and body mass index is 29 kg/m².

What is the most appropriate action with regards to his anti-diabetic medication?

	No changes to medication
	Exenatide
	Repaglinide
	Pioglitazone
	> Sitagliptin

Pioglitazone should be avoided in this man due to his history of heart failure. He would also not fit the NICE criteria for exenatide.

A HbA1c of 7.7% (61 mmol/mol) should be improved if possible. Sitagliptin may be added to metformin and gliclazide.

How well did you know this?
1
Not at all
2
3
4
5
Perfectly
32
Q

T2DM - Insulin Therapy - Example Question

A

A 56-year-old man with a history of type 2 diabetes managed with Humalog mix 30 and metformin 1g BD comes to the clinic for review. HbA1c is currently 57. He has troublesome hypoglycaemia episodes in the late afternoon and early mornings and wants to know what to do about it. On examination his blood pressure is 132/82 mmHg, his pulse is 72 beats per minute and regular. His body mass index is 32 kg/m².

Investigations

Na+	139 mmol/l
K+	4.9 mmol/l
Urea	5.1 mmol/l
Creatinine	94 µmol/l
HbA1c	57 mmol/mol

Which of the following is the most appropriate next step in his management?

	Continue current regimen, eat a snack mid afternoon and before bed time
	Reduce the dose of metformin
	Reduce the dose of mixed insulin
	> Switch to a basal bolus regimen
	Switch to Humalog mix 20

This person’s HbA1c is above target at 57 mmol/mol or 7.4%. In spite of this, he is still suffering from hypoglycaemia episodes as a result of delivering his insulin as a twice a day regimen. Moving him to basal bolus will allow splitting up of the short acting component of his insulin requirements, and therefore reduce the risk of hypoglycaemia at the end of each dosing period.

Reducing the dose of his mixed insulin may drive a further rise in his HbA1c, as would reducing the dose of metformin. Switching to Humalog mix 20 will increase the proportion of long-acting insulin and worsen the risk of hypoglycaemia. Eating a snack will drive weight gain and worsen his HbA1c.

How well did you know this?
1
Not at all
2
3
4
5
Perfectly
33
Q

Insulin SE

A

Insulin therapy: side-effects

Hypoglycaemia
patients should be taught the signs of hypoglycaemia: sweating, anxiety, blurred vision, confusion, aggression
conscious patients should take 10-20g of a short-acting carbohydrate (e.g. a glass of Lucozade or non-diet drink, three or more glucose tablets, glucose gel)
every person treated with insulin should have a glucagon kit for emergencies where the patient is not able to orally ingest a short-acting carbohydrate
patients who have frequent hypoglycaemic episodes may develop reduced awareness. If this develops then allowing glycaemic control to slip for a period of time may restore their awareness
beta-blockers reduce hypoglycaemic awareness

Lipodystrophy
typically presents as atrophy of the subcutaneous fat
can be prevented by rotating the injection site

How well did you know this?
1
Not at all
2
3
4
5
Perfectly
34
Q

T2DM - Example Question

A

A 64-year-old man is reviewed in clinic. He has a history of ischaemic heart disease and was diagnosed with type 2 diabetes mellitus around 12 months ago. At this time of diagnosis his HbA1c was 7.6% (60 mmol/mol) and he was started on metformin which was titrated up to a dose of 1g bd. The most recent bloods show a HbA1c of 6.8% (51 mmol/mol). He has just retired from working in the IT industry and his body mass index (BMI) today is 28 kg/m². His other medication is as follows:

Atorvastatin 80mg on
Aspirin 75mg od
Bisoprolol 2.5 mg od
Ramipril 5mg od

What is the most appropriate next step?

	Add sitagliptin
	> Make no changes to his medication
	Add glimepiride
	Add pioglitazone
	Add exenatide

Since the publication of the 2015 guidelines, NICE recommend we only add another drug if the HbA1c has risen to >= 58 mmol/mol (7.5%) at this stage.

How well did you know this?
1
Not at all
2
3
4
5
Perfectly
35
Q

T2DM and SGLT-2 Inhibitors - Example Question

A

You are seeing a 50-year-old lady with type 2 diabetes mellitus in the outpatient clinic. She has a past medical history of gastritis, moderate left ventricular dysfunction and chronic obstructive pulmonary disease. She is currently on metformin and gliclazide. Since last review she has gained 5kg in weight and her HbA1c has deteriorated to 70 mmol/mol from 62 mmol/mol. Body mass index today in clinic is 33 kg/m².

Recent blood tests are as follows:

Na+ 141 mmol/l
K+ 3.9 mmol/l
Urea 6 mmol/l
Creatinine 140 µmol/l

She was unable to previously tolerate liraglutide due to nausea and vomiting. What would be the best alteration to her therapy?

	> Empagliflozin (SGLT-2 inhibitor)
	Add insulin
	Add pioglitazone
	Increase dose of metformin
	Increase dose of gliclazide

SGLT inhibitors have the advantage of improving glycaemic control/HbA1c and having beneficial effects on weight. This is because their mode of action is independent of insulin release. They act upon the SGLT-2 receptors in the kidney and lead to increased loss of glucose in the urine.

How well did you know this?
1
Not at all
2
3
4
5
Perfectly
36
Q

T2DM - Mx: Example Question

A

You are reviewing a 57 year-old gentleman in the diabetes outpatient clinic. He has type 2 diabetes mellitus and is currently taking metformin 850mg three times a day and gliclazide 80mg once daily.

On further questioning he admits having frequent hypoglycaemic episodes at night that distress him as he lives alone. His BMI is calculated at 30.3 kg/m², HbA1c 7.8% (62 mmol/mol) and his co-morbidities include congestive cardiac failure.

How would you change his diabetic treatment?

 Stop gliclazide, start insulin 
 Add exenatide 
 Add sitagliptin to current regimen 
 Stop gliclazide, start pioglitazone 
 > Stop gliclazide, start sitagliptin 

The NICE guidance on the management of type 2 diabetes mellitus:
•This gentleman has been started on metformin and a sulphonyurea as first line therapy.
•He is having frequent hypoglycaemic episodes secondary to his sulphonylurea and yet control remains poor, HbA1c 7.8% (62 mmol/mol)
•Pioglitazone is contraindicated due to his congestive cardiac failure.
•A DPP-4 inhibitor such as sitagliptin would be a sensible option, the sulphonylurea should be stopped to prevent hypoglycaemia

How well did you know this?
1
Not at all
2
3
4
5
Perfectly
37
Q

Hypoglycaemia - Example Question

A

A 24-year-old nurse presents after collapsing on a night shift. His blood glucose is measured at being 1.4 mmol/l. His blood pressure at the time was noted to be 115/82 mmHg. He has no palpitations and had not bitten his tongue or become incontinent during the episodes. He was shaken afterwards, although did not have memory loss and stated he had not tripped over anything. He also said he has had five of these episodes over the last two weeks.

Blood tests are sent off and unremarkable except for a low-normal C-peptide level and markedly raised insulin level.

Which of the following is the most likely diagnosis of his multiple episodes of collapse?

 Sulphonylurea misuse 
 > Insulin misuse 
 Alcohol misuse 
 Retroperitoneal sarcoma 
 Insulinoma 

Hyperinsulinaemia in the absence of raised C-peptide points towards the diagnosis of insulin abuse. Elevation of C-peptide, when combined with hyperinsulinaemia suggests sulphonylurea abuse. To rule this out it may be appropriate to assay levels of commonly used sulphonylureas in urine. Insulinomas are a more rare cause of repeated hypoglycaemic episodes.

How well did you know this?
1
Not at all
2
3
4
5
Perfectly
38
Q

Hypoglycaemia Causes

A

Hypoglycaemia

Causes
•insulinoma - increased ratio of proinsulin to insulin
•self-administration of insulin/sulphonylureas
•liver failure
•Addison’s disease
•alcohol

Other possible causes in children
•nesidioblastosis - beta cell hyperplasia

How well did you know this?
1
Not at all
2
3
4
5
Perfectly
39
Q

T2DM and Indications for Exenatide - Example Question

A

A 52-year-old lorry driver was referred to the secondary care diabetes mellitus by his GP with poorly controlled diabetes. He was diagnosed with type 2 diabetes mellitus six years ago. Unfortunately, he has not engaged with lifestyle interventions and his diabetes has been poorly controlled since diagnosis. He has since developed diabetic nephropathy and proliferative retinopathy. Other than diabetes he has a past medical history comprising of ischaemic heart disease, hypertension, hypercholesterolaemia, osteoarthritis and gout. At the point of referral he was prescribed aspirin 75mg OD, ramipril 10mg OD, simvastatin 40mg ON, naproxen 250mg BD, co-codamol 8/500 2tabs QDS, lansoprazole 30mg OD, metformin 500mg TDS, gliclazide 80mg BD and pioglitazone 30mg OD. He smoked 20 cigarettes per day and consumed 15 units of alcohol per week.

Examination revealed an obese gentleman with a BMI of 38 kg/m. His blood pressure was 148/88 mmHg. Cardiovascular examination revealed the presence of normal heart sounds and a JVP of 3cm. Examination of the respiratory, abdominal and neurological systems was unremarkable. Investigations reveal the following results:

Na+ 136 mmol/l 
K+ 5.1 mmol/l 
Urea 14.1 mmol/l 
Creatinine 148 µmol/l 
Total cholesterol 5.1 mmol/l 
HDL cholesterol 1.3 mmol/l 
HbA1c 68 mmol/mol (8.4%) 

What is the next best step management step?

>  Commence exenatide 
 Commence sitagliptin 
 Commence insulin glargine 
 Increase dose of pioglitazone 
 Commence orlistat 

This gentleman fulfills the NICE guidelines criteria for exenatide therapy. He is obese with poorly controlled diabetes despite multiple oral hypoglycaemic agents and is suffering from other morbidities relating to his weight. Exenatide may help facilitate weight loss and does not cause hypoglycaemia. Insulin is the other option here, but it may lead to weight gain. Furthermore, owing to his occupation as a lorry driver he would require a period of close observation of his blood sugar levels prior to being allowed to resume HGV driving. Exenatide is therefore the best option.

How well did you know this?
1
Not at all
2
3
4
5
Perfectly
40
Q

DKA - Features

A

Diabetic ketoacidosis

Precipitating Factors:
Infection
MI
Missed Insulin doses

Diabetic ketoacidosis may be a complication existing type 1 diabetes mellitus or be the first presentation, accounting for around 6% of cases. Whilst DKA remains a serious condition mortality rates have decreased from 8% to under 1% in the past 20 years.

The most common precipitating factors of DKA are infection, missed insulin doses and myocardial infarction

Features
•abdominal pain
•polyuria, polydipsia, dehydration
•Kussmaul respiration (deep hyperventilation)
•Acetone-smelling breath (‘pear drops’ smell)

How well did you know this?
1
Not at all
2
3
4
5
Perfectly
41
Q

DKA - Diagnostic Criteria

A

Diagnostic criteria

American Diabetes Association (2009)
Key points
•glucose > 13.8 mmol/l
•pH < 7.30
•serum bicarbonate <18 mmol/l
•anion gap > 10
•ketonaemia

Joint British Diabetes Societies (2013)
Key points
•glucose > 11 mmol/l or known diabetes mellitus
•pH < 7.3
•bicarbonate < 15 mmol/l
•ketones > 3 mmol/l or urine ketones ++ on

How well did you know this?
1
Not at all
2
3
4
5
Perfectly
42
Q

DKA - Mx

A

Management
•fluid replacement: most patients with DKA are deplete around 5-8 litres. Isotonic saline is used initially. Please see an example fluid regime below.
•insulin: an intravenous infusion should be started at 0.1 unit/kg/hour. Once blood glucose is < 15 mmol/l an infusion of 5% dextrose should be started
•correction of hypokalaemia

How well did you know this?
1
Not at all
2
3
4
5
Perfectly
43
Q

JBDS Example of Fluid Regime for DKA

A

JBDS example of fluid replacement regime for patient with a Systolic BP on admission 90mmHg and over

Fluid > Volume

  1. 9% sodium chloride 1L 1000ml over 1st hour
  2. 9% sodium chloride 1L with potassium chloride 1000ml over next 2 hours
  3. 9% sodium chloride 1L with potassium chloride 1000ml over next 2 hours
  4. 9% sodium chloride 1L with potassium chloride 1000ml over next 4 hours
  5. 9% sodium chloride 1L with potassium chloride 1000ml over next 4 hours
  6. 9% sodium chloride 1L with potassium chloride 1000ml over next 6 hours

Please note that slower infusion may be indicated in young adults (aged 18-25 years) as they are at greater risk of cerebral oedema.

JBDS potassium guidelines

Potassium level in first 24 hours (mmol/L )> Potassium replacement in mmol/L of infusion solution
Over 5.5 = Nil
3.5-5.5 = 40
Below 3.5 = Senior review as additional potassium needs to be given

How well did you know this?
1
Not at all
2
3
4
5
Perfectly
44
Q

Complications of DKA and its treatment

A

Complications of DKA and its treatment
•gastric stasis
•thromboembolism
•arrhythmias secondary to hyperkalaemia/iatrogenic hypokalaemia
•iatrogenic due to incorrect fluid therapy: cerebral oedema, hypokalaemia, hypoglycaemia
•acute respiratory distress syndrome
•acute kidney injury

How well did you know this?
1
Not at all
2
3
4
5
Perfectly
45
Q

DKA - Example Question

A

A 30-year-old male with background of type one diabetes mellitus presents with abdominal pain and shortness of breath. Investigations confirm he has diabetic ketoacidosis. Which one of the following investigations would suggest a discussion for possible intensive care admission?

 Lactate 3 mmol/L 
 Bicarbonate level 19 mmol/L 
 pH 7.27 
 White cell count 30 x 10^9/L 
>  Potassium 3.4 mmol/L 
Parameters indicate severe diabetic ketoacidosis:
•pH < 7
•Blood ketone > 6 mmol/L
•Bicarbonate < 5 mmol/L
•Anion gap >16 mmol/l
•Potassium < 3.5 mmol/L on admission
•Tachycardia or bradycardia
•Systolic blood pressure <90 mmHg
•Oxygen saturation <92% on air
•GCS < 12
How well did you know this?
1
Not at all
2
3
4
5
Perfectly
46
Q

DKA - Mx: Example Question

A

A 31-year-old female presents with a 2-day history of abdominal pain, a 5-day history of diarrhoea and vomiting and reduced appetite. She is a known type one diabetic with background diabetic retinopathy and stage 3 chronic kidney disease. She usually takes 32 units lantus at night and variable doses of Novomix with meals however due to her poor appetite she has not taken these for 2 days. On examination, she looks unwell. The airway is patent and chest is clear. Respiratory rate is 26/min with normal oxygen saturations on air. Pulse is 120/min and thready with a capillary refill of 3 seconds centrally. Blood pressure is 103/45 mmHg with a temperature of 36.7oC. Abdomen is generally tender without guarding. Capillary blood glucose is 26 and ketones are 4.9. Arterial blood gas is as follows:

pH 7.32 
pO2 11.6 kPa 
pCO2 3.32 kPa 
Bicarbonate 14 mmol/l 
Base Excess -6.5 mmol/l 
Lactate 2.1 mmol/l 

The patient is currently being fluid resuscitated appropriately. What form of insulin therapy would you advise?

Dose of novomix now and increase usual lantus by 8 units
Variable rate i.v insulin, continue all s/c insulins
Recommence normal s/c insulin regimen
> Fixed rate i.v insulin, continue lantus
Fixed rate i.v insulin, stop all s/c insulin

This patient has a compensated metabolic acidosis and despite her reasonably preserved pH, a bicarbonate of 14 suggests significant metabolic acidosis. In the presence of high blood ketones and hyperglycaemia, the diagnosis is diabetic ketoacidosis (DKA). Fluid therapy and potassium supplementation is a key part of DKA management but will not be dealt with in this question. Current recommendations advise fixed rate insulin with the addition of 10 % dextrose when capillary blood glucose drops below 14. The aim is to correct the underlying ketoacidosis brought about by a hypoinsulinaemic state. Patients should be continued on their long acting insulin but short/medium term insulins should be held. This ensures a smooth transfer from fixed rate to usual insulin regimen when the patient has clinically and biochemically improved.

Guidelines for the management of DKA can be found here:
https://www.bsped.org.uk/clinical/docs/DKAManagementOfDKAinAdultsMarch20101.pdf

How well did you know this?
1
Not at all
2
3
4
5
Perfectly
47
Q

Diabetic Complications: Diabetic Nephropathy

A

Diabetic nephropathy

Basics
commonest cause of end-stage renal disease (ESRD) in the western world
33% of patients with type 1 diabetes mellitus have diabetic nephropathy by the age of 40 years
approximately 5-10% of patients with type 1 diabetes mellitus develop (ESRD)

The pathophysiology is poorly understood, however:
changes to the haemodynamics of the glomerulus is thought to be key, which leads to an increased glomerular capillary pressure
histological changes include: basement membrane thickening, capillary obliteration, mesangial widening. Nodulular hyaline areas develop in the glomuli - Kimmelstiel-Wilson nodules

How well did you know this?
1
Not at all
2
3
4
5
Perfectly
48
Q

Risk Factors for Developing Diabetic Nephropathy

A

Risk factors for developing diabetic nephropathy

Modifiable	
Hypertension
Hyperlipidaemia
Smoking
Poor glycaemic control
Raised dietary protein	

Non-modifiable
Male sex
Duration of diabetes
Genetic predisposition (e.g. ACE gene polymorphisms)

How well did you know this?
1
Not at all
2
3
4
5
Perfectly
49
Q

T1DM and Gastroparesis Mx: Example Question

A

A 39 year-old man presents to his GP for an annual review of his type 1 diabetes. His main complaints over the last year are having several episodes of vomiting after meals and chronic constipation, as well as having loss of sensation on both of his legs up to his knees and some sensory loss in his fingertips. On further questioning, you establish there has been no weight loss or haematamesis. On examination, his HbA1c is 72 mmol/mol, blood pressure is 138/88 mmHg and his pulse is regular and 84 beats per minute. Neurological examination demonstrates a lack of proprioception up to the ankle joint and loss of sensation as described above.

What is the most appropriate symptomatic treatment for the gastrointestinal symptoms described above?

	Lansoprazole
	Omeprazole
	> Metoclopramide
	Mirtazapine
	Cyclizine

The most likely diagnosis in this scenario is gastroparesis, caused by the type 1 diabetes. Metoclopramide is the most appropriate treatment as it is a pro-kinetic antiemetic, although erythromycin and domperidone can also be used as alternatives.

How well did you know this?
1
Not at all
2
3
4
5
Perfectly
50
Q

Diabetes and HTN - Example Question

A

A 54-year-old gentleman is reviewed prior to discharge. He was admitted three days ago with an infective exacerbation of COPD requiring nebulisers and IV antibiotics but he did not need non-invasive ventilation or ITU admission. He has responded well to treatment and his wheeze is completely gone. His cough is also much improved. He has a background of type 2 diabetes mellitus, gout, diabetic retinopathy and hypertension. He raises concerns about his blood pressure as the nurses have told him it is persistently around 150mmHg systolic. He monitors blood pressure at home and normally his readings are much better. You reassure him that it is likely because his ramipril was held on admission. What should his blood pressure be less than during his next GP review?

	> 130/80mmHg
	135/85mmHg
	140/80mmHg
	140/90mmHg
	130/90mmHg

The correct answer is 130/80mmHg. This is a patient with hypertension, type 2 diabetes and diabetic retinopathy. For a patient with hypertension without any other comorbodities, a target of less than 140/90mmHg is recommended. If the patient also has diabetes then a target of less than 140/80mmHg is recommended, whilst if there is diabetes and organ damage (kidney, eye or cerebrovascular) then a target of less than 130/80mmHg is recommended.

Source:
‘Type 2 diabetes in adults: management.’ NICE Guideline [NG28]. National Institute of Care and Health Excellence, July 2016.

How well did you know this?
1
Not at all
2
3
4
5
Perfectly
51
Q

T2DM Mx: Example Question

A

A 55-year-old man presents to the endocrine clinic. He was diagnosed five years ago with type 2 diabetes and is struggling to control his sugars. He is currently taking:

Metformin 1g BD
Glicazide 160mg BD
Sitagliptin 100mg OD

He is a bus driver and struggles to control his weight with his hectic shifts. Current BMI is 34 kg/m².

Investigations:

Serum creatinine 120 µmol/L (60-110)
Haemoglobin A1c 66 mmol/mol (8.2%)

What would be the most appropriate next step?

	Canagliflozin
	Glibenclamide
	Increase metformin
	Stop sitagliptin and add insulin
	> Stop sitagliptin and add exenatide

Given the NICE guidance the most appropriate step would be to start this patient on exenatide. This patient is already of metformin, glicazide and sitagliptin and the blood sugar levels are not under control. His BMI is under 35 but insulin would make ‘it much more difficult for you to do your job’.

How well did you know this?
1
Not at all
2
3
4
5
Perfectly
52
Q

Monitoring of Gestational DM postpartum: Example Question

A

A 28-year-old lady is diagnosed with gestational diabetes in her first pregnancy. Her fasting blood glucose is 5.9mmol/l and blood glucose after oral glucose tolerance test (OGTT) is 8.2mmol/l. Blood glucose control during pregnancy is achieved with diet, exercise and metformin. She gives birth to a healthy child at 39 weeks. A fasting blood glucose at day 1 post-partum is 5.2mmol/l.

Which of the following statements is correct with respect to follow-up monitoring for diabetes?

OGTT 6-13 weeks postpartum
> Fasting blood glucose test 6-13 weeks postpartum
No routine follow up unless further pregnancy
HbA1c 6-13 weeks postpartum
Annual fasting blood glucose checks only

Women with gestational diabetes whose glucose returns to normal after birth need a postnatal glucose check 6-13 weeks postpartum to stratify their risk of developing diabetes in the future. NICE recommends that this is a fasting blood glucose. Further follow up will depend on the result of this postnatal check. Even if postnatal glucose is less than 6mmol/l, annual fasting glucose checks are still recommended thereafter.

How well did you know this?
1
Not at all
2
3
4
5
Perfectly
53
Q

T2DM Mx: Example Question

A

A 62-year-old man comes to the Emergency department with nausea and vomiting which has steadily worsened over the past 2-3 weeks. He had Type 2 diabetes for the past 7 years and is currently treated with metformin, sitagliptin and empagliflozin. He tells you he has lost some 5kg in weight over the past month. On examination his blood pressure is 110/65 mmHg, his pulse is 85 beats per minute and regular. Emergency blood testing reveal elevated ketones and a glucose of 12.2 mmol/l.

Which of the following is the most appropriate way to manage his glucose control?

Add liraglutide
Add long-acting insulin
Change the empagliflozin for liraglutide
> Change the empagliflozin for long-acting insulin
Stop the metformin

Given the duration of Type 2 diabetes and the fact that patient has lost weight in the past month, the possibility that he is insulinopenic is raised. In this situation, calorie loss and metabolic disturbance can be exacerbated by the use of SGLT-2 inhibitors and patients may present as here, with euglycaemic ketoacidosis. The SGLT-2 inhibitor should be withdrawn, and given he is insulinopaenic, long-acting insulin added.

In this situation the empagliflozin must be withdrawn, therefore options including adding liraglutide and long-acting insulin are incorrect. GLP-1 agonists such as liraglutide work less well in patients who are relatively insulinopenic, so liraglutide is incorrect. Stopping the metformin won’t remove the cause of ketosis, the empagliflozin.

http://care.diabetesjournals.org/content/38/9/1638

How well did you know this?
1
Not at all
2
3
4
5
Perfectly
54
Q

Normoglycemic Ketoacidosis: Cx of SGLT-2 Inhibitors

A

Normoglycaemic ketoacidosis

Normoglycaemic ketoacidosis is increasingly recognised as a consequence of SGLT-2 inhibitor therapy in type 2 diabetes, and now more and more in patients with type 1 diabetes who are prescribed the drug off-license. The reason is thought to be that spilling of glucose into the urine leads to a reduction in plasma glucose and patients back off on insulin dose. They are then underdosed with respect to insulin, and SGLT-2 inhibitors also promote a rise in glucagon, which drives lipid oxidation and can further worsen the risk of ketoacidosis.

The problem can be avoided potentially by reducing the dose of the SGLT-2 inhibitor as patients reach blood glucose target.

How well did you know this?
1
Not at all
2
3
4
5
Perfectly
55
Q

Normoglycaemic Ketoacidosis - Example Question

A

A 30-year-old woman with a history of type 1 diabetes presents to the Emergency department with nausea and vomiting coupled with increased urinary frequency over the past 3 days. She has been progressively losing weight and reducing her insulin dose after starting empagliflozin prescribed to help her lose weight and reduce glucose fluctuations. She also admits to taking a Chinese herbal remedy for weight control. Blood pressure is 100/70 mmHg, pulse is 88 beats per minute. pH is 7.25, glucose is 8.1 mmol/l, urine testing reveals ketones +++

Which of the following is the most likely diagnosis?

	Empagliflozin related nephrotoxicity
	Hyperosmolar non-ketotic state
	> Normoglycaemic ketoacidosis
	Starvation ketoacidosis
	Urinary sepsis

Hyperosmolar non-ketotic state is associated with marked hyperglycaemia and no ketosis. Starvation ketosis results in the formation of urinary ketones, but pH remains normal.

http://www.fda.gov/Drugs/DrugSafety/ucm446845.htm

How well did you know this?
1
Not at all
2
3
4
5
Perfectly
56
Q

Diabetes and HTN - Example Question

A

A 48-year-old man who was diagnosed with type 2 diabetes mellitus presents for review. During his annual review he was noted to have the following results:

Total cholesterol	5.3 mmol/l
HDL cholesterol	1.0 mmol/l
LDL cholesterol	3.1 mmol/l
Triglyceride	1.7 mmol/l
HbA1c	6.4%

A QRISK2 score is calculated showing that he has a 12% 10-year risk of developing cardiovascular disease. His current medication is metformin 500mg tds. According to recent NICE guidelines, what is the most appropriate action?

	Simvastatin 40mg on
	Lifestyle advice, repeat lipid profile in 3 months
	Atorvastatin 40mg on
	> Atorvastatin 20mg on
	Increase his metformin slowly to 1g tds

NICE recommend the following when considering the use of statins in patients with type 2 diabetes mellitus:

Offer atorvastatin 20 mg for the primary prevention of CVD to people with type 2 diabetes who have a 10% or greater 10-year risk of developing CVD.

How well did you know this?
1
Not at all
2
3
4
5
Perfectly
57
Q

Hyperosmolar Hyperglycaemic State (HHS)

A

Hyperosmolar hyperglycaemic state

Hyperosmolar hyperglycaemic state (HHS) is confirmed by:
Dehydration
Osmolality >320mosmol/kg
Hyperglycaemia >30 mmol/L with pH >7.3, bicarbonate >15mmolL and no significant ketonenaemia <3mmol/L

How well did you know this?
1
Not at all
2
3
4
5
Perfectly
58
Q

HHS - Example Question

A

A 75-year-old man with a history of high blood pressure, type 2 diabetes and hypercholesterolaemia was admitted to the emergency department with confusion. His daughter states that this has come on slowly over the last week and prior to this he had no memory problems. He currently takes metformin, ramipril, amlodipine and atorvastatin.

On examination, he smells strongly of urine and his mucous membranes appear dry. His abbreviated mental test score is 7 out of 10 and he is oriented in person but not in place or time. His heart rate is 95 per minute and his blood pressure is 105/62 mmHg. His chest is clear and has a soft ejection systolic murmur which does not radiate. His jugular venous pressure is not visible and he has mild ankle oedema. He has diffuse tenderness in the lower abdomen with no peritonism and normal bowel sounds. He has no focal neurology.

Investigation results are as follows:

Chest x-ray: Clear lung fields.

Urine dip:

Glucose	+++
Blood	+
Protein	+
Leucocytes	+
Nitrites	+
Ketones	+

Venous blood gas:

pH	7.43
BE	- 1.5 mmol/l
HCO3	23 mmol/l
Glucose	34 mmol/l
Lactate	2.5 mmol/l

Full blood count:

Hb 120 g/l
Platelets 445 * 109/l
WBC 13 * 109/l

Renal function:

Na+	151 mmol/l
K+	5 mmol/l
Urea	10 mmol/l
Creatinine	137 µmol/l
Glucose	32 mmol/l
Ketones	2 mmol/l

Which would be the most appropriate initial resuscitation measure?

	0.45% saline
	> 0.9% saline
	Fixed rate insulin and 0.9% saline
	Hartmann's
	Sliding scale insulin and 0.9% saline

This gentleman has hyperosmolar hyperglycaemic state (HHS), likely precipitated by urinary tract infection and his pre-existing diabetes.

According to the Joint British Diabetes Society Guidelines for HHS, 0.9% saline is the recommended initial resuscitation fluid, aiming for 3-6 litres positive at 12 hours. This should only be switched to 0.45% saline if osmolality is declining despite positive fluid balance. Fixed rate insulin should only be added if glucose fails to fall with fluid.

Reference: Joint British Diabetes Societies Inpatient Care Group. The management of hyperosmolar hyperglycaemic state (HHS) in adults with diabetes. 2012.

59
Q

HHS Mx: Example Question

A

A 66-year-old male was admitted with agitation and confusion, worsening over the past 1 week. His past medical history includes hypertension, ischaemic heart disease and chronic back pain. His daughter noticed that he had lost about 1 stone in weight (currently weighs 71 kg), has been more tired over the last month and that he has been drinking a lot more water. This was associated with the development of urinary incontinence.

On examination, his heart rate was 108 beats/min, blood pressure was 95/42 mmHg, saturations were 94% on air and respiratory rate was 20/min. He is confused, with a Glasgow Coma Scale of 14 and appeared dehydrated.

Blood results are as follows:

Na+	125 mmol/l
K+	5.0 mmol/l
Urea	18 mmol/l
Creatinine	180 µmol/l
Blood glucose	34 mmol/l

Venous blood gas was done and showed the following:

pH	7.32
pCO2	4.6 kPa
pO2	6.1 kPa
HCO3	17mmol/l
BE	-3.6 mmol/l

Which is the most important treatment?

	> Intravenous 0.9% sodium chloride
	10 units of human actrapid stat
	Start insulin sliding scale at 6 units/hr
	Calcium gluconate
	Intravenous 1.8% sodium chloride

Characteristic features of hyperosmolar hyperglycaemic state (HHS) includes:
high osmolality, often 320 mosmol/kg or more
high blood glucose, usually 30 mmol/L or more
severely dehydrated and unwell.
without significant hyperketonaemia or acidosis

HHS typically occurs in the elderly and is often the first presentation of Type 2 Diabetes Mellitus.

Using the blood results, osmolality can be calculated with the formula 2(Na+K) +glucose+urea

Goals of treatment include:
Normalise the osmolality
Replace fluid and electrolyte losses
Normalise blood glucose
Prevention of complications: Arterial or venous thrombosis/cerebral oedema

Fluid replacement must commence first; an initial insulin bolus of 0.15 U per kg may be given once infusions are underway. Fluid replacement alone with 0.9% sodium chloride solution will result in falling blood glucose. Insulin treatment prior to adequate fluid replacement may result in cardiovascular collapse as water moves out of the intravascular space, with a resulting decline in intravascular volume.

Patients with HHS are often exquisitely sensitive to insulin and require much lower doses than in diabetic ketoacidosis (DKA).The recommended insulin dose is a fixed rate intravenous insulin infusion (FRIII) given at 0.05 units per kg per hour (e.g. 4 units/hr in an 80 kg man) is used

Beware of rapid correction of hyponatraemia, may lead to cerebral pontine myelinolysis

Source: http://www.diabetes.org.uk/Documents/Position%20statements/JBDS-IP-HHS-Adults.pdf

60
Q

HHS - What you need to know

A

Characteristic features of hyperosmolar hyperglycaemic state (HHS) includes:
high osmolality, often 320 mosmol/kg or more
high blood glucose, usually 30 mmol/L or more
severely dehydrated and unwell.
without significant hyperketonaemia or acidosis

HHS typically occurs in the elderly and is often the first presentation of Type 2 Diabetes Mellitus.

Using the blood results, osmolality can be calculated with the formula 2(Na+K) +glucose+urea

Goals of treatment include:
Normalise the osmolality
Replace fluid and electrolyte losses
Normalise blood glucose
Prevention of complications: Arterial or venous thrombosis/cerebral oedema

Fluid replacement must commence first; an initial insulin bolus of 0.15 U per kg may be given once infusions are underway. Fluid replacement alone with 0.9% sodium chloride solution will result in falling blood glucose. Insulin treatment prior to adequate fluid replacement may result in cardiovascular collapse as water moves out of the intravascular space, with a resulting decline in intravascular volume.

Patients with HHS are often exquisitely sensitive to insulin and require much lower doses than in diabetic ketoacidosis (DKA).The recommended insulin dose is a fixed rate intravenous insulin infusion (FRIII) given at 0.05 units per kg per hour (e.g. 4 units/hr in an 80 kg man) is used

Beware of rapid correction of hyponatraemia, may lead to cerebral pontine myelinolysis

61
Q

T2DM Mx: Example Question

A

A 50-year-old man who drives a heavy goods vehicle comes to the diabetes clinic for review. Current medication for diabetes includes metformin 1g BD and gliclazide 160mg BD. His blood sugars have steadily increased over the past few months and his most recent HbA1c is 72 mmol/mol. He has increased in weight by 5kg over the past 12 weeks which he puts down to work, (driving his lorry for excessive periods). On examination his blood pressure is 155/88 mmHg, pulse is 75 beats per minute and regular. Abdomen is soft and non-tender, body mass index is 37 kg/m².

Investigations:

Na+ 140 mmol/l
K+ 5.0 mmol/l
Urea 7.1 mmol/l
Creatinine 110 µmol/l

Which of the following is the most appropriate next step?

	Add basal insulin
	Add mixed insulin
	Add pioglitazone
	Switch gliclazide to linagliptin
	> Switch gliclazide to liraglutide

A number of factors come into play with respect to management of this patient’s blood glucose. His occupation as a lorry driver precludes medication which may significantly increase his risk of hypoglycaemia. This effectively rules out insulin initiation unless he is prepared to accept any impact on his job. Insulin would also promote further weight gain and salt and water retention, and is therefore less desirable as an option. Pioglitazone promotes weight gain and fluid retention, so again is not really an option.

This leaves us with the two switching scenarios. Switching to liraglutide is preferred, because in trials GLP-1 receptor agonists have been shown to reduce HbA1c by similar levels to basal insulin, without increasing the risk of hypoglycaemia, and promote approximately 3% weight loss over a 6 month period. DPPIV inhibitors such as linagliptin are potentially less effective in reducing HbA1c than sulphonylureas and linagliptin is therefore not an option here.

62
Q

MODY - Maturity Onset Diabetes of the Young

A

MODY

Maturity-onset diabetes of the young (MODY) is characterised by the development of type 2 diabetes mellitus in patients < 25 years old. It is typically inherited as an autosomal dominant condition. Over six different genetic mutations have so far been identified as leading to MODY.

It is thought that around 1-2% of patients with diabetes mellitus have MODY, and around 90% are misclassified as having either type 1 or type 2 diabetes mellitus.

MODY 3
60% of cases
due to a defect in the HNF-1 alpha gene

MODY 2
20% of cases
due to a defect in the glucokinase gene

Features of MODY
typically develops in patients < 25 years
a family history of early onset diabetes is often present
ketosis is not a feature at presentation
patients with the most common form are very sensitive to sulfonylureas, insulin is not usually necessary

63
Q

MODY - Example Question

A

A 24 year-old man presents with a five week history of increasing thirst and frequency of urinating. The GP suspects diabetes and performs two fasting blood tests on separate days which reveal blood glucose results of 8.7 mmol/l and 9.2 mmol/l. Urinalysis does not detect any ketones or protein in the urine. The patient’s mother had a diagnosis as type 1 diabetes at the age of 22 and his maternal grandfather and aunt also have type 1 diabetes. Due to the family history, the patient’s c-peptide is measured and found to be consistently high on two occasions.

Given the likely diagnosis, what is the most appropriate first treatment for managing this condition?

	> Gliclazide
	Metformin
	Pioglitazone
	Insulin
	Sitagliptin

The most likely diagnosis in this scenario is maturity onset diabetes of youth, due to the persistently raised c-peptide and strong family history. It normally presents in early adulthood and 1/3 of cases can be treated with oral hypoglycaemics, such as sulphonylureas. It is therefore important to see the response to sulphonylureas first before commencing insulin therapy.

64
Q

T2DM - Mx: Example Question

A

A 62-year-old woman with a history of type 2 diabetes comes to the clinic for review. She has a history of mild cardiac failure managed with ramipril and bisoprolol. Her current medication for diabetes is metformin 1g BD. On examination her blood pressure is 122/82 mmHg, pulse is 80 beats per minute and regular. There are bilateral basal crackles on auscultation of the chest, and pitting oedema of both ankles. Her body mass index is elevated at 33 kg/m².

HbA1c 73 mmol/mol
Creatinine 82 µmol/l

Which of the following is the most appropriate next step for managing glucose control?

	> Empagliflozin
	Glipizide
	Liraglutide
	Pioglitazone
	Saxagliptin

This patient has moderate obesity with a BMI of 33, cardiac failure, and a poorly controlled HbA1c at 33. Out of the available options empagliflozin, an SGLT-2 inhibitor is the most appropriate. As well as lowering blood glucose, SGLT-2 inhibition also has a diuretic like effect, lowering blood pressure and reducing fluid retention. Empagliflozin has also been shown to reduce cardiovascular mortality, and this patient is at high risk of an event.

Glipizide is inappropriate as it may drive further weight gain in a patient who is already obese, saxagliptin is recognised to cause heart failure, and pioglitazone has also been shown in clinical trials to cause fluid retention. This leaves liraglutide as the other potential option, which doesn’t significantly impact on fluid balance and according to NICE is only indicated at BMI of 35 and above.

65
Q

Intolerance to Metformin - Example Question

A

A 53-year-old woman comes for review in the general medical clinic. She was diagnosed with type 2 diabetes mellitus six months ago after having developed fatigue and polyuria. She also has hypothyroidism but no other comorbidities. She was started on metformin 500mg twice daily struggled to cope due to gastrointestinal side effects such as diarrhoea. What is the most appropriate action?

	Reduce to metformin 500mg once daily
	Change to dipeptidyl peptidase-4 inhibitor
	> Trial of modified release metformin
	Change to sulfonylurea
	Change to pioglitazone

The correct answer trial of modified release metformin. NICE guidelines advise to offer standard release metformin as the first-line treatment for type 2 diabetes and to gradually increase the dose to minimise the risk of gastrointestinal side effects. If gastrointestinal side effects are not tolerated, then a trial of modified release metformin would be appropriate. If metformin is not tolerated at all then a dipeptidyl peptidase-4 inhibitor, sulfonylurea or pioglitazone would be indicated.

Source:

‘Type 2 diabetes in adults: management.’ NICE guideline [NG28]. The National Institute for Health and Care Excellence, December 2015.

66
Q

T1DM and Statins and QRISK2 - Example Question:

A

You are review a 38-year-old woman with type 1 diabetes mellitus in clinic. Her diabetes is currently controlled with a basal-bolus regime. She takes no other medication apart from citalopram 20mg od for depression. She was diagnosed with type 1 diabetes at the age of 13 years. Her most recent bloods show

Na+ 142 mmol/l
K+ 3.9 mmol/l
Urea 4.9 mmol/l
Creatinine 79 µmol/l

Total cholesterol 4.4 mmol/l
HDL cholesterol 1.2 mmol/l
LDL cholesterol 1.8 mmol/l
Triglyceride 1.3 mmol/l

Urine dip: No protein or blood

What is the most appropriate management with regards to lipid modification?

	Start atorvastatin 10mg on
	> Start atorvastatin 20mg on
	Start atorvastatin 40mg on
	Perform a QRISK2 assessment
	Reassure her that lipid modification therapy is not required at this stage

NICE specifically state that we should not use QRISK2 for type 1 diabetics. Instead, the following criteria are used:
older than 40 years, or
have had diabetes for more than 10 years or
have established nephropathy or
have other CVD risk factors

This patient has had diabetes for 25 years so we should start atorvastatin 20mg on.

67
Q

Diabetes and QRISK2

A

NICE specifically state that we should not use QRISK2 for type 1 diabetics. Instead, the following criteria are used:
older than 40 years, or
have had diabetes for more than 10 years or
have established nephropathy or
have other CVD risk factors

68
Q

Target HbA1c in a patient post pancreatectomy - Example Question

A

A 42-year-old man has been diagnosed with diabetes following a resection for chronic pancreatitis. He has been initiated on insulin therapy and comes to the clinic to discuss a target HbA1c. His GP has been striving for a target HbA1c of 42, although the patient has pushed back, saying that this is resulting in significant hypoglycaemia. Clinical examination is unremarkable, his blood pressure is 142/82 mmHg, pulse is 80 and regular. His chest is clear. He is thin with a midline scar on his abdomen, his body mass index is 21 kg/m².

Which of the following is an appropriate target HbA1c for him?

	31
	37
	42
	48
	> 53

It’s important to remember in diabetes related to pancreatic resection, both alpha cells producing glucagon, and beta cells producing insulin, are removed in the pancreatectomy. This reduces the intensity of any counter regulatory response to hypoglycaemia and thus impacts on prospects of recovery and increases the severity of individual events. For this reason a more lax HbA1c target is instigated for patients who have a history of pancreatectomy, of 53. HbA1cs of 31 and 37 are actually within the normal range.

69
Q

Diabetes Mellitus and Ramadan

A

Diabetes mellitus: Ramadan

We know that type 2 diabetes mellitus is more common in people of Asian ethnicity and a significant proportion of those patients in the UK will be Muslim. The BMJ published an excellent and comprehensive review of this issue in 20101.

It is important that we can give appropriate advice to Muslim patients to allow them safely observe their fast. This is particularly important from 2014 as Ramadan is due to fall in the long days of the summer months for several years henceforth.

Clearly it is a personal decision whether a patient decides to fast. It may however be worthwhile exploring the fact that people with chronic conditions are exempt from fasting or may be able to delay fasting to the shorter days of the winter months. It is however known that many Muslim patients with diabetes do not class themselves as having a chronic/serious condition which should exempt them from fasting. Around 79% of Muslim patients with type 2 diabetes mellitus fast Ramadan2.There is an excellent patient information leaflet from Diabetes UK and the Muslim Council of Britain which explores these options in more detail.

If a patient with type 2 diabetes mellitus does decide to fast:
they should try and and eat a meal containing long-acting carbohydrates prior to sunrise (Suhoor)
patients should be given a blood glucose monitor to allow them to check their glucose levels, particularly if they feel unwell
for patients taking metformin the expert consensus is that the dose should be split one-third before sunrise (Suhoor) and two-thirds after sunset (Iftar)
expert consensus also recommends switching once-daily sulfonylureas to after sunset. For patients taking twice-daily preparations such as gliclazide it is recommended that a larger proportion of the dose is taken after after sunset
no adjustment is needed for patients taking pioglitazone

  1. Management of people with diabetes wanting to fast during Ramadan BMJ 2010;340:c3053
  2. Salti I et al. Results of the Epidemiology of Diabetes and Ramadan (EPIDIAR) study. Diabetes Care 2004;27:2306-11.
70
Q

T2DM and Metformin during Ramadan

A

During Ramadan, one-third of the normal metformin dose should be taken before sunrise and two-thirds should be taken after sunset

Example Question:

A 65-year-old man who is known to have type 2 diabetes mellitus presents for advice. He is a Muslim and is considering fasting for Ramadan. His diabetes is currently controlled with a combination of diet and metformin 500mg tds. Looking at his records the last HbA1c was 6.4% (46 mmol/mol). If he decides to fast during Ramadan, what is the most appropriate advice to give regarding his metformin?

Metformin should be stopped
Metformin 1.5g after sunset
> Metformin 500mg before sunrise, 1g after sunset
Metformin 500mg after sunset
Metformin 1g before sunrise, 500mg after sunset
71
Q

Choice of Anti-hypertensive in Diabetes regardless of age - Example Question

A

You are reviewing in clinic a 67-year-old man who has type 2 diabetes. His glycaemic control is reasonable with metformin therapy; the latest HbA1c is 54 mmol/mol (7.1%). A few weeks ago he was noted to have a clinic blood pressure reading of 152/90 mmHg. A 24 hour blood pressure monitor was requested. The report shows his average blood pressure was 142/88 mmHg. What is the most appropriate course of action?

Do nothing for now, monitor his blood pressure regularly
> Start an ACE inhibitor
Start a calcium channel blocker
Repeat the 24 hour blood pressure monitor in 4-8 weeks time
Request an ultrasound of his kidneys ACE inhibitors are first-line for hypertension in diabetics, irrespective of the patients age

This patient has stage 1 hypertension as defined by NICE. He should however be treated because he has underlying diabetes.

The first-line treatment for a patient aged > 55 years is a calcium channel blocker. However, in patients with diabetes ACE inhibitors are used first-line due to their renoprotective effect.

72
Q

Resolution of DKA - Example Question

A

A 24 year old male presented with diabetic ketoacidosis, 12 years after initially diagnosed with type 1 diabetes. He was admitted with the following arterial blood gas:

pH 7.14
Bicarbonate 4 mmol/l

Blood glucose 26 mmol/L
Ketones 9 mmol/L

He was started on an intravenous fixed rate insulin infusion and intravenous fluid rehydration regime in a high dependency setting. At day 3 after the onset of treatment, nursing ask you review the latest blood gas from his arterial line. His fixed rate insulin continues to run. Which of the following blood gases would confirm resolution of his diabetic ketoacidosis?

pH 7.32, bicarbonate 13, blood glucose 7, ketones 0.3 mmol/L
pH 7.29, bicarbonate 9, blood glucose 15, ketones 0.9 mmol/L
pH 7.34, bicarbonate 14, blood glucose 12, ketones 0.9 mmol/L
pH 7.17, bicarbonate 8, blood glucose 14, ketones 1.2 mmol/L
> pH 7.31, bicarbonate 15, blood glucose 11, ketones 0.5 mmol/L

According to the Joint British Diabetes Societies Inpatient Care Group, resolution of DKA is defined as:
pH >7.3
bicarbonate > 15 mmol/L
ketones < 0.6 mmol/L

Note that blood glucose is NOT part of the criteria in defining resolution of DKA. On resolution, insulin infusion should be stopped and the patient should be restarted on their previous insulin regime with input from the diabetic team to consider readjustment of doses. The underlying cause of DKA should also be sought. In newly diagnosed patients, subcutaneous insulin glargine (lantus) should be consider until further diabetic review.

73
Q

Resolution of DKA - Criteria

A

According to the Joint British Diabetes Societies Inpatient Care Group, resolution of DKA is defined as:
pH >7.3
bicarbonate > 15 mmol/L
ketones < 0.6 mmol/L

74
Q

T2DM Mx: Example Question

A

A 58 year-old man with type 2 diabetes mellitus presents to his GP for his annual review. His diabetes is currently being treated with metformin 500mg twice daily and gliclazide 40mg once daily. He complains that recently he has had several episodes of hypoglycaemia and this has been affecting him at work where his job is as a construction worker. His most recent HbA1c was 66 mmol/mol. Apart from the diabetes, he also has a past medical history of stage 2 heart failure, hypertension and mild chronic obstructive pulmonary disease.

What is the most appropriate treatment change?

	Stop gliclazide and start exenatide
	> Stop gliclazide and start sitagliptin
	Commence daily insulin
	Stop gliclazide and start glibenclamide
	Stop gliclazide and start pioglitazone

This patient’s hypoglycaemic attacks is most likely due to the sulfonylurea he is currently on. NICE guidance recommends that if the patient is suffering hypoglycaemic side effects from the sulfonylurea they can be commenced on a combination of metformin and a glitazone or metformin and a DPP-4 inhibitor. However, due to this patient’s heart failure, pioglitazone is not suitable, therefore a combination of metformin and sitagliptin is the most appropriate option. Glibenclamide has long-acting hypoglycaemic effects so would not be appropriate in this scenario.

75
Q

Diabetic Nephropathy - Mx

A

Diabetic nephropathy: management

Screening
all patients should be screened annually
albumin:creatinine ratio (ACR) in early morning specimen
ACR > 2.5 = microalbuminuria

Management
dietary protein restriction
tight glycaemic control
BP control: aim for < 130/80 mmHg
benefits independent of blood pressure control have been demonstrated for ACE inhibitors (ACE-i) and angiotensin II receptor blockers (A2RB). Combinations of ACE-i and A2RB are not commonly used anymore following the ON-TARGET trial which showed worse outcomes for patients on dual blockade
control dyslipidaemia e.g. Statins
76
Q

Diabetic Nephropathy Mx: Example Question

A

You are the endocrinology registrar in the diabetic clinic. You see a 45 year old female with type 2 diabetes mellitus. She is generally well in herself. She currently takes metformin 500 mg bd and has no other regular medications. She has good hypoglycaemia awareness and rarely gets hypoglycaemic episodes. Blood sugars on home monitoring range between 5 and 9.

Routine blood tests show:

Hb	131 g/l
Plt	131 x10^9/l
WCC	13.1 x10^9/l
Na+	131 mmol/l
K+	3.1 mmol/l
Urea	3.1 mmoll
Creatinine	31 µmol/l
eGFR	79 ml/kg/1.73m²
HbA1c	46 mmol/mol (6.4%)

Urinalysis: PRO+, GLUC +, nil else.

Urinary ACR (albumin:creatinine ratio) is 3.6 mg/mmol.

On examination, her BMI is 21 kg/m², she has good peripheral sensation, fundi look normal. Chest is clear to auscultation, heart sounds are normal with no murmurs and her abdomen is soft and non tender.

Her heart rate is 72 beats per minute and regular, blood pressure is 124/75 mmHg, respiratory rate is 20/min, oxygen saturations are 98% on air and temperature 36.8 degrees.

What medication should be commenced in this lady for her ongoing management?

	Commence gliclazide 80mg BD
	> Commence ramipril 1.25mg OD
	Increase metformin to 500 mg TDS
	Commence candesartan 2 mg OD
	Commence antibiotics for suspected UTI

The only abnormalities in this case are an unexplained raised white cell count and a raised urinary albumin:creatinine ratio (>3.5 mg/mmol for women or >2.5 mg/mmol for men). There are no symptoms or other tests to suggest a cause for the elevated white cell count and giving antibiotics for a suspected UTI would not be advised given the absence of nitrites and leukocytes on urine dipstick. The patient does not need referring to the renal team at the moment as the renal function is fine.

The patient’s diabetic control is good with a Hba1c of 46 mmol/mol and does not therefore need an increased dose of oral hypoglycaemic agent. This leaves us with two options - a) and b). B is the correct answer due to the patients microscopic proteinuria - an ace inhibitor would be commenced despite the normal blood pressure as a renal protective agent as an elevated ACR is a predictor of deterioration in renal function due to diabetic nephropathy in the future. There is no need to add gliclazide into this gentlemans medications for improved glycaemic control as his HbA1c is within acceptable limits for current treatment to continue as-is according to NICE guidelines.

77
Q

T2DM with CKD - Example Question

A

A 71-year-old gentleman presents to clinic for review. He has recently been diagnosed with type two diabetes following screening by his GP. His HbA1c has not responded to dietary changes that he was advised on. He has a past medical history of bladder cancer for which he had chemotherapy, hypertension, macular degeneration, eczema and chronic kidney disease. His baseline eGFR is 28ml/min/1.73m2.

Blood tests:
Na+	139 mmol/l
K+	4.4 mmol/l
Urea	6.2 mmol/l
Creatinine	214 µmol/l

What is the most appropriate medication to start?

	Metformin
	> Sulfonylurea
	Pioglitazone
	Insulin
	GLP-1 mimetic

The correct answer is a sulfonylurea. This is a patient with new type two diabetes and initial treatment would normally be metformin, but since the eGFR is <30ml/minute/1.73m2 metformin is contraindicated. The next step would be either a a dipeptidyl peptidase4 inhibitor, pioglitazone or a sulfonylurea. However, pioglitazone should not be offered to a patient with a history of bladder cancer. Therefore that leaves sulfonylurea as the correct answer. Insulin and GLP-1 mimetic could be used further down in the treatment algorithm.

Source:

‘Type 2 diabetes in adults: management.’ NICE guideline [NG28]. The National Institute for Health and Care Excellence, December 2015.

78
Q

MODY - Example Question

A

A 36-year-old woman is reviewed 3 months post-partum. During the pregnancy she was diagnosed with gestational diabetes. Following the delivery her glycaemic control has failed to improve and she has been diagnosed as having type 2 diabetes mellitus. She is only slightly overweight (body mass index 27.1 kg/m²) and you are worried about missing maturity onset diabetes of the young (MODY) or type 1 diabetes. Which one of the following is most suggestive of MODY?

Ketosis during periods of hyperglycaemia
> Family history of early onset diabetes mellitus
A history of polycystic ovarian syndrome
Lack of response to sulphonylureas
A history of autoimmune disease

MODY is inherited in an autosomal dominant fashion so a family history is often present

79
Q

Necrobiosis Lipoidica - Example Question

A

A 45 year-old woman presents to her GP with an ulcer on her left shin. It has been present for the past 4 months and gradually got bigger. It is tender and yellow-brown in colour and there appears to be another smaller lesion that is growing next to the bigger one. She has a past medical history of multiple sclerosis, of which she is currently in remission phase and type 1 diabetes mellitus. Her only regular medication includes insulin.

What is the most appropriate treatment?

	Flucloxacillin
	> Topical corticosteroids
	TED stockings
	Oral corticosteroids
	Topical tacrolimus

The most likely diagnosis is necrobiosis lipoidica, a rare granulomatous skin disorder which can affect the shin of insulin dependent diabetics. Skin biopsy can confirm the diagnosis, although it is often done clinically and topical steroids are a useful treatment, along with injectable corticosteroids and camouflage creams.

80
Q

T2DM Mx: Example Question

A

A 78-year-old man with type 2 diabetes mellitus is reviewed in the diabetes clinic. He is currently taking metformin 1g bd. He also has a history of hypertension and hypothyroidism. His HbA1c one year ago was 44 mmol/mol (6.2%). The most recent test is reported as 46 mmol/mol (6.4%). What is the most appropriate next step in management?

	Increase dose of metformin
	Add glimepiride
	Add sitagliptin
	Add pioglitazone
	> Make no changes

This man has acceptable glycaemic control, both in terms of NICE guidance and more recent evidence looking at the harms of overzealous glycaemic control. No changes should therefore be made for now.

81
Q

Gestational Diabetes - Ix - Example Question

A

A 27-year-old woman who is 11 weeks pregnant comes for review. This is her second pregnancy. During her first pregnancy she was diagnosed with gestational diabetes which resolved following the birth of her son. What is the most appropriate management at this stage?

> Perform an oral glucose tolerance test
Advise on a diabetic diet and start metformin at 20 weeks
Arrange a fasting glucose
Arrange a HbA1c test
Advise on a diabetic diet and start insulin at 20 weeks

The oral glucose tolerance test remains the investigation of choice for gestational diabetes

82
Q

T2DM Mx - Example Question

A

A 79-year-old lady with a body mass index of 31 kg/m² attends clinic complaining of increasing tiredness. She is found to have a random glucose reading of 15.5 mmol/L and a recent HbA1c of 7.5%. She is currently on metformin 1g twice daily and recently stopped gliclazide due to hypoglycaemic episodes.

Which drug therapy should be added next to further control her glucose readings?

	> Sitagliptin
	Insulin
	Glimepiride
	Liraglutide
	Canagliflozin

This patient has poorly controlled type 2 diabetes mellitus given her random blood glucose reading of 15 mmol/L and HbA1c of 7.5%. Her current treatment with metformin is suitable given her current BMI in keeping with obesity. However, she is having significant hypoglycaemic episodes with gliclazide (sulfonylurea).

Therefore the next best agent to add is sitagliptin (DPP 4 inhibitor). Sitagliptin is recommended as a second agent alongside metformin (first line) when blood glucose control is inadequate (i.e. HbA1c > or equal to 6.5%) and a sulfonylurea is contraindicated or not tolerated (e.g. hypoglycaemia). Sitagliptin would be preferable in this instance to a thiazolidinedione (pioglitazone) as further weight gain would be undesirable.

Glimepiride is a longer acting second generation sulfonylurea drug and would not be suitable due to existing problems with hypoglycaemia.

Liraglutide (long-acting glucagon-like peptide 1 receptor agonist) can be considered in patients with a raised BMI if combinations of metformin, sulfonylurea, DPP 4 inhibitor and a thiazolidinedione are not tolerated or either drug is contraindicated.

Canagliflozin (sodium glucose co-transporter inhibitor) can be given as second-line agent with metformin if a sulfonylurea is contraindicated or not tolerated. It can also be given as a third agent in addition to metformin and a sulfonylurea or thiazolidinediones. The reported side effects of this drug are hypoglycaemia when used in combination with insulin or a sulfonylurea, vaginal candidiasis, urinary tract infection, polyuria and urinary frequency. Insulin should be considered with uncontrolled hyperglycaemia despite treatment with dual or triple oral hypoglycaemic agents.

83
Q

DKA - Mx with Fluids and K+ - Example Question

A

A 23 year old woman with known type 1 diabetes mellitus is brought to the emergency department resus area unwell. On taking her history she mentions that she has had a cough for the past three days with fevers, and today she developed vague abdominal pain. A blood gas take on admission shows:

pH	7.21
pO2	14.8 kPa
pCO2	3.1 kPa
Bicarbonate	14 mEq/L
Base excess	-10.5 mmol/L
Na+	145 mmol/L
K+	5.3 mmol/L
Glucose	40.1 mmol/L
Lactate	2.3 mmol/L
Urinalysis:
Leucocytes	-
Blood	-
Glucose	+++
Ketones	++++

The patient has been given 500mls of 0.9% saline as a stat bag, followed by a further 1L of 0.9% saline over 60 minutes, and her observations are as follows:

Respiratory rate	25 breaths/minute
Saturations	99% on 2L
Temperature	37.7ºC
Blood pressure	106/67 mmHg
Heart Rate	102 beats/minute

The nurses weigh her at 60 Kg
Her lab results are pending

Which fluid and insulin regimen would be the most appropriate at her current stage?

1L of 0.9% Normal Saline with 20mmol of KCl added given over 2 hours + IV insulin at 6 units per hour.
> 1L of 0.9% Normal Saline with 40mmol of KCl added given over 2 hours + IV insulin at 6 units per hour.
1L of 0.9% Normal Saline with 20mmol of KCl added given over 2 hours + Sliding scale IV insulin based on capillary blood sugar readings
1L of 0.9% Normal Saline with 20mmol of KCl added given over 2 hours + 1L of 1.26% sodium bicarbonate solution over 8h + IV insulin at 6 units per hour.
1L of 0.9% Normal Saline with 40mmol of KCl added given over 4 hours + IV insulin at 6 units per hour.

Answering this question requires knowledge of the up to date guidelines on treating diabetic ketoacidosis (see link below). There are a few key points to the new guidelines which make this question easy to answer by a process of elimination.

First is potassium replacement in fluid which is based on the patients current potassium

Current potassium level mmol/L > Potassium replacement mmol/L of infusion solution
>5.5 = Nil
3.5-5.5 = 40 mmol
<3.5 = senior review

Second is the rate at which fluids should be given in the early part of management, and is based on the initial blood pressure (where a systolic blood pressure of 90mmHg is a cut off for getting senior help). When the systolic blood pressure is over 90 mmHg the fluid replacement after the initial 500ml stat is as follows:

Fluid > Volume

  1. 9% sodium chloride 1L 1000ml over 1st hour
  2. 9% sodium chloride 1L with potassium chloride 1000ml over next 2 hours
  3. 9% sodium chloride 1L with potassium chloride 1000ml over next 2 hours
  4. 9% sodium chloride 1L with potassium chloride 1000ml over next 4 hours
  5. 9% sodium chloride 1L with potassium chloride 1000ml over next 4 hours
  6. 9% sodium chloride 1L with potassium chloride 1000ml over next 6 hours

Finally the guidelines regarding IV bicarbonate are clear:
Adequate fluid and insulin therapy will resolve the acidosis in DKA and the use of bicarbonate is not indicated. The acidosis may be an adaptive response as it improves oxygen delivery to the tissues by causing a right shift of the oxygen dissociation curve. Excessive bicarbonate may cause a rise in the CO2 partial pressure in the cerebrospinal fluid (CSF) and may lead to a paradoxical increase in CSF acidosis. In addition, the use of bicarbonate in DKA may delay the fall in blood lactate: pyruvate ratio and ketones when compared to intravenous 0.9% sodium chloride infusion. There is some evidence to suggest that bicarbonate treatment may be implicated in the development of cerebral oedema in children and young adults.

This leaves only one possible option.

Full guidance:
https://www.diabetes.org.uk/Documents/About%20Us/What%20we%20say/Management-of-DKA-241013.pdf

84
Q

DKA in T2DM - Example Question

A

A 72 year old female presents with 5 days of generally decline and a recent productive urinary tract infection, treated with oral antibiotics in the community by the GP. She is known to be a type two diabetic, diagnosed 28 years ago and insulin dependent for the past 6 years. She is normally on 46 units lantus, 23 units TDS novorapid. On examination, she is not orientated in time or place, GCS 14/15. She has no focal neurology, chest and cardiovascular ausculation are unremarkable. You demonstrate suprapubic tenderness on deep palpation but abdomen is other soft and nontender, bowel sounds are present. She appears extremely dehydration: her mucous membranes are dry, peripheries cool with capillary refill time of 4 seconds and JVP +1 cm above the angle of Louis. Her blood sugar is 31mmol/L and a venous blood gas demonstrates pH 7.22, lactate 2 mmol/l, ketones 5 mmol/l. A urine dip is awaited. What is the most likely diagnosis?

Hyperglycaemic hyperketotic state
Hyperglycaemia secondary to poor medical complicance during recent acute illness
Urosepsis secondary to inadequately treated UTI
> Diabetic ketoacidosis
Dehydration secondary to poor oral intake

The patient is acidotic with ketones >3 mmol/l demonstrated, on a background of known insulin dependence. Although she is known to be type 2 diabetic, it should be remembered that both types of diabetics can present as DKA, particularly advanced T2 DM who produce little to no endogenous insulin and are hence unable to shut down ketogenesis. Treatment should be as per DKA protocols, with intravenous fluids, fixed rate insulin infusion @ 0.1 unit/kg/hour, thromboprophylaxis, broad spectrum antibiotics and appropriate K+ replacement with insulin1.

85
Q

HIV Related Diabetes and Metformin - Example Question

A

A 60-year-old man presented with history of excessive urination especially during night which disturbs his sleep, excessive thirst and tiredness for the past 2 months. He was a known HIV positive patient for which he was on highly active anti-retroviral therapy (HAART) since 6 months. Clinical examination was within normal limits. HIV related diabetes mellitus was suspected by the treating physician. His blood investigations are as follows-

Hb	150 g/l
Platelets	150 * 109/l
WBC	7 * 109/l
Fasting blood glucose	10 mmol/l
Post prandial blood glucose	13.9 mmol/l
Serum creatinine	130 umol/L

He was planned to be started on metformin therapy. Which of the following investigations is important to be checked before starting metformin in this patient?.

	Liver enzymes
	> Venous lactate
	Serum triglycerides
	Serum albumin
	Serum ammonia

Patients with HIV are at increased risk of lactic acidosis and hence it is prudent to check lactate levels before starting metformin in such patients.Lactic acidaemia with no or mild symptoms was detected in 8% to 21% of patients receiving at least 1 nucleoside reverse transcriptase inhibitor, versus 0% to 1% of patients receiving no antiretroviral therapy. Symptomatic lactic acidaemia is less common (occurring in about 1.5%2.5%).Monitoring for lactic acidosis is recommended in the initial few months of metformin therapy. Patients with impaired renal function(serum creatinine>1.5 mg/dl) or with a venous lactate level more than twice the normal level prior to treatment should not be started on metformin.

This patient has an elevated serum creatinine but not >1.5mg/dl. So, he should get a venous lactate estimation before being started on metformin therapy considering the evidences so far.

86
Q

DKA - Example Question

A

A 19-year-old man is brought by ambulance to the emergency department. The patient himself was too unwell to provide a coherent history but his mother reports that he had been very unwell with loss of appetite and abdominal pain for the past two days. She had also noted a significant weight loss in her son over the past six months and that he had been unusually fatigued over the same time period. The patient had no previous family history although his older sister had recently been diagnosed with pernicious anaemia.

Examination showed the patient to be tachypnoeic and tachycardic with a central capillary refill time of six seconds. Respiratory and cardiovascular examination was otherwise unremarkable. Abdominal examination revealed some inconsistent tenderness without obvious signs of localised peritonism.

Please see below for results of initial investigations.

Urea	10.6 mmol / L
Creatinine	134 micromol / L
Sodium	149 mmol / L
Potassium	5.8 mmol / L
Fingerpick blood glucose	42 mmol / L
Fingerpick blood ketones	7.5 mmol / L
Venous blood gas (room air)
pH	7.09
PCO2	16 mmHg (reference 32-43)
PO2	52 mmHg (reference 70-100)
Bicarbonate	10.9 mmol / L (reference 20.0-26.0)
Chloride	107 mmol / L (reference 99-108)
Lactate	4.9 mmol / L

Portable chest x-ray: poor quality film secondary overlying artefact; no free air under diaphragm, lung fields clear, no pneumothorax.

Electrocardiogram: sinus tachycardia at 129; normal intervals and axis; QRS morphology normal; T wave morphology normal.

What is the correct immediate management for this patient?

	1000 mg calcium gluconate
	10 mL 8.4 % sodium bicarbonate
	> 1000 mL 0.9 % saline
	1000 ml 0.45 % saline
	Intravenous insulin bolus

The patient has clear biochemical evidence of diabetic keto-acidosis, most likely representing a first presentation of type 1 diabetes mellitus.

Correct immediate management is to give a litre of 0.9 % saline to restore intravascular volume while preventing a rapid change in extracellular osmolality. For subsequent IV fluids, The Joint British Diabetes Society recommends ongoing use of 0.9 % saline, in contrast to the American Diabetes Association that recommends 0.45 % saline if sodium level normal or high.

An initial bolus dose of intravenous insulin is not recommended after a randomisation controlled trial evidence found no benefit. A weight based insulin infusion of 0.1 units / mg / Kg is recommended.

Bicarbonate infusion are not routinely recommended in the treatment of DKA, although may be considered in patients with severe acidosis (pH < 7) and associated life threatening effects of acidaemia.

Given the absence of ECG changes of hyperkalaemia in this patient, an infusion of calcium gluconate is not required. In fact, the patient is likely to be more at risk of hypokalaemia as treatment for DKA progresses.

Misra S, Oliver N. Diabetic ketoacidosis in adults. BMJ 2015;351:h5660.

87
Q

Metformin and Ramadan

A

A 65-year-old Muslim man with type II diabetes on metformin (500mg three times a day) comes to your endocrine clinic. He is about to start fasting between sunrise and sunset for Ramadan. He will typically eat a light meal before sunrise (Suhoor) and a large meal at sunset (Iftar).

How would you advise this gentleman in respect to his metformin prior to the large meal at sunset?

Change to a sulphonylurea
Stop metformin
Take 1.5g metformin before the large meal at sunset
Take 500mg metformin before the large meal at sunset
> Take 1g metformin before the large meal at sunset

This patient will still require metformin and therefore it is not appropriate to stop it.

The total daily dose of metformin should be split:
One-third before sunrise (Suhoor) (500mg)
Two-thirds after sunset (Iftar) (1g)

There is no need to change to a sulphonylurea if his blood glucose levels are controlled on metformin alone.

88
Q

T2DM and when to offer Insulin - Example Question

A

A 50-year-old gentleman presents to review in the endocrine clinic. He was normally taking metformin and gliclazide, but over the last six months has also been prescribed a GLP-1 mimetic. He was started on a GLP-1 mimetic because of poor diabetic control and concerns about starting insulin. He does not want to start insulin because he is a truck driver and worried about losing his driving license. He had previously tried the combination of metformin, gliclazide and pioglitazone, but this had also failed to control his HbA1c.

Over the last six months his HbA1c has reduced from 81mmol/ mol to 80mmol/ mol, and he has lost two kg in weight. What is the most appropriate action?

Continue current treatment and review in one year
Continue current treatment and review in six months
Change to metformin, gliclazide and pioglitazone
Offer to add on insulin
> Stop GLP-1 mimetic and offer insulin treatment

The correct answer is to stop GLP-1 mimetic and offer insulin treatment. This is a patient with diabetes whose diabetic control has not improved even when on a combination of metformin and two other drugs. He was then started on GLP-1 mimetic therapy, but nice advice that this should only be continued if after six months HbA1c reduces by at least 11 mmol/mol, which has not happened. Therefore continuing the current treatment is incorrect. Insulin should not be combined with a GLP-1 mimetic.

Source:

‘Type 2 diabetes in adults: management.’ NICE guideline [NG28]. The National Institute for Health and Care Excellence, December 2015.

89
Q

T2DM and HTN - Example Question

A

A 70-year-old woman with a history of type 2 diabetes mellitus and hypertension is reviewed in clinic. You can see from the records their is no evidence of diabetic retinopathy, chronic kidney disease or cardiovascular disease.

Her current medication is as follows:
simvastatin 40mg on
ramipril 10mg od
amlodipine 5mg od
metformin 1g bd

Recent blood results are shown below:

Na+	142 mmol/l
K+	4.4 mmol/l
Urea	7.2 mmol/l
Creatinine	86 µmol/l
HbA1c	45 mmol/mol (6.3%)

Urine dipstick shows no proteinuria. Her blood pressure today in clinic is 134/76 mmHg.

What is the most appropriate course of action?

	Add gliclazide
	Increase amlodipine
	Increase ramipril
	Add losartan
	> No changes to medication required

Her diabetic control is good - NICE do not advocate changing treatment at this stage unless the HbA1c is >= 6.5%.

As she has no complications from her diabetes the blood pressure target is < 140/80 mmHg. No changes are therefore required to her antihypertensive regime.

90
Q

Alcohol induced DKA - Example Question

A

A 19-year-old woman presents to the emergency department drowsy and vomiting. She is accompanied by a friend who tells you she has been out drinking all day and has been vomiting for the last few hours.

Her speech is slurred and confused, she opens her eyes in response to her name and pushes you away in response to a painful stimulus. Heart rate is 100 beats per minute and regular, blood pressure is 100/60 mmHg, capillary glucose is 18 mmol/L, and a urine dip shows pH: 4, blood: trace, ketones: +++, protein: trace, nitrites: negative and leukocytes: negative.

Chest x-ray: Normal

Venous blood gas:

pH	7.27 (7.35-7.45)
Bicarbonate	10mmol/L (22-26)
Base excess	-10 (-2 to +2)
Sodium	135 mmol/L (137-144)
Potassium	2.9 mmol/L (3.5-4.9)
Chloride	99 mmol/L (95-107)

Serum Glucose: 21 mmol/L

What is the most important initial intervention?

	Fixed rate intravenous insulin infusion (FRIII)
	Intravenous calcium gluconate
	> Intravenous fluids
	Sliding scale insulin
	Urgent CT head

This woman fits the diagnostic criteria for diabetic ketoacidosis (DKA):

Ketonaemia > 3.0mmol/L or significant ketonuria (more than 2+ on standard urine sticks)
Blood glucose > 11.0mmol/L or known diabetes mellitus
Bicarbonate (HCO3- ) < 15.0mmol/L and/or venous pH < 7.3

Diabetes UK guidance states that the most important initial therapeutic intervention in DKA is appropriate fluid replacement followed by insulin administration.

The key benefits of fluid resuscitation in this context include:

Recovery of circulatory volume
Clearance of ketones and therefore improvement of acidosis
Correction of electrolyte imbalance

Weight based fixed rate intravenous insulin infusion (FRIII) is now the recommended mode of insulin administration in DKA, over sliding scale.

DKA is a complication of type i diabetes (but can rarely complicate type ii diabetes). It can be the first presentation of type i diabetes, result from poor diabetic control or be precipitated by another factor such as infection.

Source:
diabetes UK, guidelines https://www.diabetes.org.uk/Documents/About%20Us/What%20we%20say/Management-of-DKA-241013.pdf

91
Q

DKA - Mx: Example Question

A

A 25-year-old woman is brought to the emergency department by ambulance after being found unwell by friends. Collateral history reported by the paramedics indicated that the patient had been unwell for 3 days with vomiting and diarrhoea. Her housemate said that the patient had been unable to eat since becoming unwell and that he did not think she had been taking her regular insulin during that time. The patient herself was too disorientated to give any history. The paramedics had found both novorapid and lantus insulin pen devices in the patients fridge.

General examination indicated a drowsy and dehydrated patient with generalised abdominal tenderness but no evidence of focal peritonism.

Please see below for selected investigation results.

Observations: blood pressure 86 / 57 mmHg; heart rate 127 beats per minute; respiratory rate 28 per minute; O2 saturations 100 % (room air); Temperature 37.1 oC.

Fingerpick blood glucose	38.2 mmol / L
Fingerpick blood ketones	8.7 mmol / L
Urea	12.5 mmol / L
Creatinine	123 micromol / L
Sodium	148 mmol / L
Potassium	3.7 mmol / L
Haemoglobin	156 g / dL
White cell count	14.3 x 10>3 / microlitre
Neutrophils	11.3 x 10>3 / microlitre
Platelets	453 x 10>3 / microlitre
Arterial blood gas (room air)
pH	7.05
PaCO2	15 mmHg (reference 32-43)
PaO2	99 mmHg (reference 70-100)
Bicarbonate	12.3 mmol / L (reference 20.0-26.0)
Chloride	111 mmol / L (reference 99-108)
Lactate	7.5 mmol / L

What is the appropriate strategy for intravenous insulin treatment in this patient?

Variable rate insulin infusion without initial bolus, converting to subcutaneous insulin once acidosis resolved
> Fixed rate insulin infusion without initial bolus, converting to subcutaneous insulin once patient is eating and drinking normally
Fixed rate insulin infusion following initial bolus, converting to subcutaneous insulin once patient is eating and drinking normally
Variable rate insulin infusion with initial bolus, converting to subcutaneous insulin once acidosis resolved
Variable rate insulin infusion without initial bolus, converting to subcutaneous insulin once ketonaemia resolved

The patient is presenting in diabetic ketoacidosis due to vomiting, dehydration and omission of prescribed insulin.

The Joint British Diabetes Society recommend an insulin infusion at rate 0.1 units / kg / h. An initial bolus of insulin is not advised due to a randomised controlled trial that found no benefit. Fixed rate insulin infusions are now preferred over titration of insulin dose against blood sugar levels (sliding scale). This is due to the fact that blood glucose may correct more quickly than ketoacidosis and so ensures adequate insulin to eradicate ketones.

There is no consensus between expert bodies as to biochemical end-point of DKA, therefore it is advised that patients are transferred onto subcutaneous insulin once they are eating and drinking normally. It is vital to ensure an overlap between the administration of intravenous and subcutaneous insulin to avoid recurrent ketogenesis.

Misra S, Oliver N. Diabetic ketoacidosis in adults. BMJ 2015;351:h5660.

92
Q

GLP-1 Mimetics and Pancreatitis: Example Question

A

A 54-year-old man with a history of type 2 diabetes is recovering on the surgical ward having suffered an episode of acute pancreatitis some 4 days earlier. Medication for glucose control includes metformin, dapagliflozin and liraglutide. On examination his blood pressure is 135/80 mmHg, pulse is 72 and regular. His body mass index is 35 kg/m². A recent HbA1c is 63 mmol/mol, renal function is reported as normal. Which of the following is the correct course of action with respect to his long term blood glucose lowering medication?

	Stop metformin
	Stop dapagliflozin
	> Stop liraglutide
	Continue usual medication
	Stop metformin and liraglutide

This patient is now stable following his episode of pancreatitis, but in view of a number of studies suggesting an association between GLP-1 agonist prescription and pancreatitis, it should be withdrawn in patients who suffer pancreatitis whilst on therapy, and not instituted in patients who have a history of the disease. For this reason liraglutide should be withdrawn.

With respect to metformin, it should potentially be discontinued during periods of increased risk of tissue hypoxia, (e.g. during the acute period of pancreatitis), but there is no indication it should be omitted over the longer term. In regards to metformin and renal impairment, guidance has recently been revised to support metformin initiation at glomerular filtration rate as low as 45ml/min.

93
Q

T2 Diabetic and Afro-Caribbean - Choice of anti-hypertensive? Example Question

A

A 42-year-old patient presents for review. He has a past medical history of type 2 diabetes mellitus. He takes metformin only and has had good glycaemic control. He is an Afro-Caribbean chemistry teacher. He has a family history of early ischaemic heart disease. His blood pressure is found to be 148/90mmHg, higher than previous visits when it was 143/89mmHg. He is keen to start treatment to help prevent heart disease. What should be offered?

	ACE inhibitor only
	> ACE inhibitor and calcium-channel blocker
	Calcium channel blocker only
	Angiotensin receptor antagonist
	Beta-blocker

The correct answer is ACE inhibitor and calcium-channel blocker. This is quite a fine-print question of managment of hypertension. The important factors to note are that he has diabetes and that he is Afro-Caribbean. The diabetes makes an ACE inhibitor a better antihypertensive choice, whilst the fact that he is Afro-Caribbean means that ACE inhibitors are less likely to be effective. In this situation, NICE guidelines suggest to start a combination of an ACE inhibitor and a calcium-channel blocker or a diuretic. Afro-Caribbean ethnicity is associated with a genetic polymorphism of ACE making ACE inhibitors less likely to be effective for controlloing blood pressure.

Source:
‘Type 2 diabetes in adults: management.’ NICE Guideline [NG28]. National Institute of Care and Health Excellence, July 2016.

94
Q

HHS - Example Question

A

A 65-year-old gentleman presents with a 3 week history of general malaise, decreased oral intake and drowsiness. He has a past medical history of ischaemic heart disease, type 2 diabetes mellitus and gastritis. He lives alone with no carers and normally mobilises independently. A concerned neighbour went in to check on him after he was not seen for a few days. On examination his mouth is dry with reduced skin turgor. Heart sounds are normal, chest is clear, abdominal palpation reveals lower abdominal tenderness. ECG shows sinus tachycardia. Urine dip shows ketones +, glucose +++.

Blood tests show:

Hb	140 g/l	
Na+	150 mmol/l
Platelets	525 * 109/l	
K+	4.2 mmol/l
WBC	14 * 109/l
Urea	13 mmol/l
Neuts	10 * 109/l	
Creatinine	160 µmol/l
Lymphs	2 * 109/l	
CRP	56 mg/l
Eosin	0.5 * 109/l		

Venous blood gas shows no signs of acidosis. Formal blood glucose is phoned back as 40 mmol/L.

What is the most important initial treatment?

	Intravenous insulin
	Low molecular weight heparin
	Antibiotics
	Subcutaneous insulin
	> Intravenous fluids

The patient is clinically and biochemically dehydrated. Intravenous fluid resuscitation may be enough to normalise blood sugars. If hyperglycaemia is not responding then insulin may be required. Assessment for precipitant cause is also important such as infection, change in medicines, cardiovascular event etc. Low molecular weight heparin should be prescribed unless any contraindications due to the increased risk of venous thromboembolism associated with this condition.

95
Q

HHS - Example Question

A

A 72-year-old male was admitted drowsy and confused. His family describe a 4-day history of shortness of breath and a productive cough. His past medical history includes type 2 diabetes mellitus, hypertension and hypercholesterolaemia. He usually takes metformin 500 mg three times daily, gliclazide 80 mg twice daily, amlodipine 5 mg daily and simvastatin 40 mg nightly. On examination he is confused with dry mucous membranes, blood pressure of 100/50 mmHg, a pulse of 110/min, a temperature of 37.6 oC and a respiratory rate of 20/min. Course crepitations were found at the right base and his pulse was thready with a capillary refill of 3 seconds; jugular venous pressure was not visible. Capillary blood glucose was found to be HI.

A venous blood sample is taken:

Hb	129 g/l	Na+	161 mmol/l
Platelets	204 * 109/l	K+	4.9 mmol/l
WBC	13.1 * 109/l	Urea	15.2 mmol/l
Neuts	11.9 * 109/l	Creatinine	97 µmol/l
Glucose	56 mmol/l	eGFR	62 mg/l
Ketones	1.9 mmol/l	HbA1c	75 mmol/mol
pH	7.35	HCO3	20 mmol/mol

What treatment would you initiate first?

	> 0.9% normal saline
	0.45 % normal saline
	Hartmann's solution
	Intravenous insulin
	5% dextrose

The underlying diagnosis is hyperosmolar hyperglycaemic state (HHS) precipitated by a lower respiratory tract infection. The diagnosis is made when a patient shows marked hyperglycaemia and hypovolaemia with a serum osmolality > 320 mosm/kg in the absence of marked ketonaemia or acidosis. This patient is clinically dry with a glucose of 56 and a calculated osmolality of 2(161+4.9) + 56 + 15 = 403 mosm/kg with mild ketonaemia.

Despite this gentlemen’s hypernatraemia, the first line fluid therapy is 0.9% normal saline as patients in HHS are sodium deplete. One litre should be administered rapidly and electrolytes then repeated to gauge potassium requirements in further fluid bags. A small initial rise in sodium is expected and this should not discourage further use of normal saline. Insulin is not routinely started immediately in the absence of significant ketonaemia as glucose will fall with fluid therapy alone. When glucose stops falling insulin may be started if glucose still remains high. Too rapid correction of osmolality i.e. with aggressive insulin and fluids in combination can precipitate cerebral oedema.

Guidelines for the management of HHS can be found below:
http://www.diabetologists-abcd.org.uk/JBDS/JBDSIPHHSAdults.pdf

96
Q

DKA and Reducing Blood Glucose - Example Question

A

A 37 year old female presents with 4 days of generally unwell and a recent dysuria. Her urine is foul smelling and dark. She is a known type 1 diabetic with a long standing subcutaneous insulin regime. Her pH on admission was 7.24, bicarbonate 8 mmol/l and blood glucose 32 mmol/l. Urinary dip leucocytes 2+, nitrites2+ and 4+ ketones. She was started on treatment for diabetic ketoacidosis with intravenous fluids and fixed rate insulin. She also has intravenous antitbiotics for a urinary source of sepsis. You are asked to review her blood sugars at 4 hours after treatment was initiated. What should be the aim in managing hyperglycaemia in a diabetic ketoacidosis patient?

Reduce blood glucose to under 14 mmol/l as quickly as possible
> Reduce blood glucose by 3mmol/l per hour
Reduce blood glucose by 6 mmol/l per hour
Aim blood glucose above 18 mmol/l
Blood glucose does not require monitoring if insulin infusion is running

The most recent guidelines by the Joint British Diabetes Societies Inpatient Care Group in September 2012 recommends reduction in blood glucose of 3 mmol/l per hour, until BM reaches 14 mmol/l, at which point 5% dextrose should be considered as the intravenous fluid of choice. Rapid glucose lowering should be avoided: the rapid flux in osmolality can result in significant cerebral oedema and resultant cerebral damage.

97
Q

HHS - Example Question

A

A 62 year old male, recently emigrated from India, presents with 5 day history of feeling generally unwell. His niece, who has accompanied him to hospital, denies a history of recent productive cough, diarrhoea or vomiting or dysuria. Her uncle had been gradually increasingly malaised over the past 5 days and not eating and drinking well. He has no known past medical history. On examination, he has dry mucous membranes and cool peripheries, his JVP is +1cm above the angle of Louis. Heart sounds, chest and abdomen are unremarkable. Urine dip and chest radiograph are awaited. His blood tests are as follows:

WBC 16 * 109/l
Neutrophils 14.8 * 109/l

Na+	152 mmol/l
K+	3.7 mmol/l
Urea	22 mmol/l
Creatinine	208 µmol/l
CRP	38 mg/l
Glucose	38 mmol/l
Ketones	2.8 mmol/l

Arterial blood gases:

pH 7.31 
PaO2 20.2 kPa
PaCO2 3.0 kPa
Bicarbonate 16 mmol/l
Lactate 4 mmol/l

What is the unifying diagnosis?

	Diabetic ketoacidosis (DKA)
	Lactic acidosis
	> Hyperosmolar hyperglycaemic state (HHS)
	Urinary tract sepsis
	Chest sepsis

This patient has presented with dehydration and non-specific symptoms and a diagnosis difficult to diagnose clinically. However, his biochemistry is diagnostic: calculation of his osmolality, (2[Na + K] + urea+ glucose) reveals an osmolality greater 371.4mosmol/kg. He is likely to present acutely with undiagnosed type 2 diabetes mellitus and a diagnosis of HHS, previously known as HONK.

There is no evidence to suggest uro or chest sepsis but an infectious underlying decompensating trigger should be considered with prescription of broad spectrum antibiotics. Although lactate is mildly raised, this is likely secondary to intravascular dehydration and hypoperfusion of internal organs. Lactic acidosis alone does not account for the full biochemical picture. Ketones are present and the patient is mild acidotic. However, be aware that neither is sufficiently significant for a diagnosis of DKA.

98
Q

Insulin Therapy and Hypoglycaemia - Example Question

A

A 78-year-old female attends the diabetes clinic. She has longstanding type 2 diabetes. Over the last few years she has become increasingly frail. Her main complaint is recurrent nausea and vomiting. Earlier this year she underwent endoscopy and gastric emptying studies which confirmed gastroparesis. She has since been started on metoclopramide which has had minimal effect on her symptoms. Her weight has decreased by 10% over the past year with a current BMI of 26 kg/m².

Her HbA1c today at the clinic is 44 IFCC mmol/l (6.2%) having been 60 IFCC mmol/l (7.6%) this time last year.

Her past medical history includes chronic kidney disease stage 3 and aortic stenosis.

Her current therapy is Humulin M3 22 units at breakfast and dinner, metformin 500mg BD, ramipril 5mg OD, bendroflumethiazide 2.5mg OD, aspirin 75 mg OD.

She lives alone and is still driving. She denies the need for carers, however, she has had 3 falls in the past month. She describes particular difficulty getting up in the morning and says her mood can often be low in the mornings.

She checks her blood sugar once daily in the morning with the following results.

Saturday	3.1 mmol/l
Sunday	4.0 mmol/l
Monday	4.1 mmol/l
Tuesday	3.2 mmol/l
Wednesday	14.6 mmol/l
Thursday	3.5 mmol/l
Friday	16.1 mmol/l

What is the correct step in the management of her diabetes?

Change Humulin M3 to a glucagon-like-peptide 1 receptor agonist
Stop metformin
Add sitagliptin
> Change Humulin M3 to 20 units in the morning and 10 units in the evening
Change Humulin M3 to 30 units once daily

This lady’s fasting blood sugars are too low. The 2 high readings raise the concern of overnight hypoglycaemia with reflex hyperglycaemia in the morning. Her recent weight loss will mean her insulin resistance will have decreased (as reflected in her decreasing HbA1c) and she will require smaller doses of insulin. Changing her insulin from a total of 44 units daily to 30 units daily offers protection against hypoglycaemia. Furthermore altering her split of insulin to the conventional 2/3rd in the morning and 1/3rd in the evening reduces the risk of overnight hypos.

The risks of tight glycaemic control in this lady vastly outweigh any benefits. She should be advised to check her sugar more regularly for the short-term and given advice regarding driving. Close follow-up with the diabetic nurses should be arranged until her hypo risk is reduced.

There is no indication for a GLP1 agonist (BMI 26) and metformin should continue (assuming eGFR remains above 30) to allow the reduction in her insulin doses to lowest effective levels.

99
Q

T2DM - Diagnosis

A

Diabetes mellitus (type 2): diagnosis

The diagnosis of type 2 diabetes mellitus can be made by either a plasma glucose or a HbA1c sample. Diagnostic criteria vary according to whether the patient is symptomatic (polyuria, polydipsia etc) or not.

If the patient is symptomatic:
fasting glucose greater than or equal to 7.0 mmol/l
random glucose greater than or equal to 11.1 mmol/l (or after 75g oral glucose tolerance test)

If the patient is asymptomatic the above criteria apply but must be demonstrated on two separate occasions.

SEE PASSMED T2DM DIAGNOSIS

In 2011 WHO released supplementary guidance on the use of HbA1c on the diagnosis of diabetes:
a HbA1c of greater than or equal to 48 mmol/mol (6.5%) is diagnostic of diabetes mellitus
a HbAlc value of less than 48 mmol/mol (6.5%) does not exclude diabetes (i.e. it is not as sensitive as fasting samples for detecting diabetes)
in patients without symptoms, the test must be repeated to confirm the diagnosis
it should be remembered that misleading HbA1c results can be caused by increased red cell turnover (see below)

Conditions where HbA1c may not be used for diagnosis:
haemoglobinopathies
haemolytic anaemia
untreated iron deficiency anaemia
suspected gestational diabetes
children
HIV
chronic kidney disease
100
Q

T2DM - Impaired Fasting Glucose and Impaired Glucose Tolerance

A

Impaired fasting glucose and impaired glucose tolerance

A fasting glucose greater than or equal to 6.1 but less than 7.0 mmol/l implies impaired fasting glucose (IFG)

Impaired glucose tolerance (IGT) is defined as fasting plasma glucose less than 7.0 mmol/l and OGTT 2-hour value greater than or equal to 7.8 mmol/l but less than 11.1 mmol/l

Diabetes UK suggests:
‘People with IFG should then be offered an oral glucose tolerance test to rule out a diagnosis of diabetes. A result below 11.1 mmol/l but above 7.8 mmol/l indicates that the person doesn’t have diabetes but does have IGT.’

101
Q

T2DM - Diagnosis: Example Question

A

A 56-year-old man with a history of hypertension presents for review. As part of his annual health check he has a U&E, HbA1c and cholesterol check done. The following results are obtained:

His blood pressure today is 128/78 mmHg. His only regular medication is ramipril 5mg od.

Na+	142 mmol/l
K+	4.6 mmol/l
Urea	5.2 mmol/l
Creatinine	88 µmol/l
Total cholesterol	5.2 mmol/l
HbA1c	45 mmol/mol (6.3%)

His 10-year QRISK2 score is 7%. What is the most appropriate action following these results?

	Start atorvastatin 20mg on
	> Arrange a fasting glucose sample
	Diagnose type 2 diabetes mellitus
	Increase the dose of ramipril
	Add amlodipine 5mg od

His QRISK2 score is < 10% so no action needs taking about his cholesterol. His blood pressure is also well controlled.

His HbA1c is on the higher side and currently resides in the pre-diabetes range (42-47 mmol/mol). A HbA1c reading cannot however be used to exclude diabetes - a fasting sample should therefore be arranged.

102
Q

HHS and Indication for Insulin: Example Question

A

A 80-year-old patient was referred to Accident and Emergency after being found unresponsive in his home. He had just completed a course of antibiotics for a chest infection. He had not been seen for the preceding 36 hours. He had a past medical history of hypertension and type two diabetes.

His medication included Metformin, Gliclazide, Humulin M3 insulin twice a day, Ramipril and Bendroflumethiazide.

His initial examination revealed. Blood pressure 104/53, heart rate 103 beats per minute, respiratory rate 24 and oxygen saturations 90% on air. He had inspiratory crackles on his left lower lung zone. He had sunken eyes, capillary refill time of four seconds and no lower limb swelling. GCS 13 out of 15.

Initial blood tests;

Hb	11.0 g/dL
WCC	21.4 *10^9/l
Platelets	189 *10^9/l
CRP	340 mg/L
Na+	149 mmol/l
K+	4.4mmol/l
Ur	28 mmol/l
Cr	180 µmol/l
Glucose	54mmol/l

ABG on air

pH	7.32
pCO2	3.7kPa
pO2	9kPa
HCO3	18 mmol/l
Lactate	2.4mmol/l

Urine dipstick analysis - ++ glucose, - WCC, - leucocytes, + ketones

The patient was treated with oxygen, intravenous antibiotics for a chest infection and prophylactic low molecular weight heparin. They were treated with the local diabetic ketoacidosis protocol with IV inulin sliding scale and IV fluids 5500ml in 24 hours.

His repeat bloods 12 hours later were;

Na+	132 mmol/l
K+	3.9 mmol/l
Ur	12 mmol/l
Cr	110 µmmol/l
Glucose	5 mmol/l
HCO3	24 mmol/l
Lactate	1.7 mmol/l
CRP	270mg/l

The patient developed a grand mal seizure. His Glasgow coma scale remained 10 an hour after the seizure.

What is the most likely cause of his neurological deterioration?

	Hypoglycaemia
	Intracranial venous sinus thrombosis
	> Cerebral oedema
	Sepsis
	Renal failure

All the answers are potentially associated with hyperosmolar hyperglycaemic coma. The current guidance advises treatment initially with normal saline intravenous rehydration. The target reduction in osmolality is 3-8 mosmol/kg an hour. Only if this target is not being met and the glucose level not reducing sufficiently is insulin to be started.

This patient developed cerebral oedema secondary to rapid reduction in serum osmolality.

103
Q

Use of Metformin in T1DM - Example Question

A

A 34-year-old man of ethnic Indian origin is reviewed in endocrinology clinic. He has type 1 diabetes. He has a twice-daily mixed insulin regime but has poor diabetic control with elevated HbA1c and high blood glucose. He wants to improve his diabetic control but is concerned about increasing his insulin dose or frequency as he is already overweight with a body mass index (BMI) of 29kg/m2.

Apart from increasing insulin, are there any other medical management options to better control his diabetes?

	No further medical treatment
	> Metformin
	Gliclazide
	Acarbose
	Pioglitazone

The correct answer is metformin. This is a patient with poorly controlled type 1 diabetes. Of importance to this case, he is also overweight and is of ethnic Indian origin. Increasing insulin doses is seldom popular due to the side effects. NICE recommends that in overweight people of Indian origin metformin is an alternative to increasing insulin and may be helpful, especially as it would not lead to further weight increase.

Source:

‘Type 1 diabetes in adults: diagnosis and management’ Clinical guideline [NG17]. The National Institute for Health and Care Excellence, August 2015.

104
Q

Initial Mx of T2DM - Example Question

A

A 54-year-old man presents to the diabetes clinic for review. He has had symptoms of polyuria, polydipsia and lethargy over the past few months, and his fasting glucose is elevated at 7.6 mmol/l. He has no history of diabetes in his family and is currently treated for hypertension and dyslipidaemia by his GP. On examination his blood pressure is 155/90 mmHg, pulse is 70 beats per minute and regular. His body mass index is 34 kg/m². Other blood tests of note include GAD+ antibodies, renal function is normal.

Which of the following is most appropriate with respect to managing his glucose control?

	Gliclazide
	Liraglutide
	> Metformin
	Sitagliptin
	Basal bolus insulin

Up to 10% of patients who are thought to have type 2 diabetes are found to also be GAD autoantibody positive. These individuals are thought to progress faster to insulin start than patients with autoantibody negative type 2 diabetes, (within 3-5 years vs 7 years on average for ‘standard’ patients).

The management of these patients is the same as for patients with autoantibody negative disease, metformin as the initial therapy of choice. Weight reduction may delay progression to insulin, therefore insulin sparing strategies, at least during the first few years after diagnosis are the preferred intervention. These may include use of SGLT-2 inhibitors or GLP-1 agonists.

Diapedia reference:
http://www.diapedia.org/type-1-diabetes-mellitus/2104458121/lada-latent-autoimmune-diabetes-of-the-adult

105
Q

DKA Initial Mx - Example Question

A

A 19-year-old with type 1 diabetes presents to the Emergency Department feeling unwell. She states she has had vomiting and diarrhoea for 2 days and has not been taking her full insulin doses as she has been off her food. Her capillary glucose is 37 mmol/l and there are 4+ ketones on urinalysis.

An arterial blood gas is performed and the results are as follows:

pH	7.12
pO2	13 kPa
pCO2	3.5 kPa
HCO3	13
Na	129 mmol/l
K	6.1 mmol/l

Which of the following is the most appropriate initial management?

	> IV 0.9% NaCl bolus
	IV 10 units actrapid + 50ml 50% dextrose
	IV 8.4% sodium bicarbonate
	Empirical IV antibiotics
	Insulin sliding scale

This is a classical presentation of diabetic ketoacidosis. While precise protocols vary, the key principals are initial fluid resuscitation with normal saline prior to starting an IV insulin infusion, and careful potassium replacement.

Low sodium is often seen and is a pseudohyponatraemia secondary to the high serum glucose.

Serum potassium derangements are common and need careful management. Potassium is driven into cells by insulin. Serum potassium levels are therefore often high on presentation while blood insulin levels are depleted. Despite this, total body potassium is low due to fluid losses and requires careful monitoring and replacement during treatment.

Source: http://www.diabetes.org.uk/Documents/About%20Us/What%20we%20say/Management-of-DKA-241013.pdf

106
Q

Process responsible for Ketone production in DKA?

A

LIPOLYSIS
Low insulin conditions seen in DKA stimulates the process of lipolysis and the production of the ketone bodies, beta-hydroxybutyrate and acetoacetate, which can be used as metabolic fuel

107
Q

Diabetes - Pathophysiology

A

T1DM:
- autoimmune disease
- antibodies against beta cells of pancreas
- HLA DR4 > HLA DR3
- Various antibodies such as islet-associated antigen (IAA) antibody and glutamic acid decarboxylase (GAD) antibody are detected in patients who later go on to develop T1DM - their prognostic significance is not yet clear
- Identical twins show a genetic concordance of 40%
- Assoc w HLA-DR3 and DR4
- Inherited in a polygenic fashion
NB - There is almost 100% concordance in identical twins and no HLA associations

108
Q

Restarting Insulin in Insulin-dependent diabetic after insulin infusion

A

Stopping an insulin infusion in the context of an insulin-dependent diabetic needs to be done with care

NB Long acting insulins should be continued throughout the duration of any insulin infusion

IF this does not happen (not an uncommon problem on the wards)
Key focus = Insulin dependent diabetics should never be without insulin as they risk precipitating a DKA
Safest way = to start long acting insulin BEFORE insulin infusion is stopped - allows long acting insulin to take effect before infusion is stopped
Fast-acting insulin e.g. Novorapid should then be restarted with next meal

109
Q

Interpretation of HbA1c

A

A num of conditions can interfere with accurate HbA1c interpretation

Lower than expected levels of HbA1c (due to reduced RBC lifespan)

  • Sickle cell anaemia
  • G6PD deficiency
  • Hereditary spherocytosis

Higher than expected levels of HbA1c (due to increased RBC lifespan)

  • Vitamin B12/Folate deficiency
  • Iron deficiency anaemia
  • Splenectomy
110
Q

T2DM Diagnosis

A

T2DM Diagnosed:
If patient is symptomatic (polyuria, polydipsia, fatigue)
- Fasting glucose >= 7.0mmol/L OR
- Random blood glucose >= 11.1mmol/L (or after 75g OGTT)

If patient is asymptomatic, above criteria still apply but must be demonstrated on 2 separate occasions

The use of HbA1c on diagnosis of DM: (NB Cannot be used in children, pregnancy or HIV)

  • HbA1c >= 6.5% (48mmol/mol) = Diagnostic
  • HbA1c < 6.5% does not exclude DM
  • If asymptomatic HbA1c >= 6.5% or 48mmol/mol must be repeated

NB: Misleading HbA1c results caused by conditions where there is increased red cell turnover

111
Q

‘Pre-Diabetes’ Diagnosis

A

HbA1c 42-47mmol/mol (6.0-6.4%)

OR
Fasting glucose 6.1 - 6.9 mol/L

112
Q

Impaired fasting glucose (IFG) and Impaired Glucose Tolerance (IGT)

A

IFG = fasting glucose >= 6.1 but < 7.0 (mol/L)

IGT = fasting plasma glucose < 7.0 AND OGTT 2hr value >= 7.8 but < 11.1

113
Q

Thiazolidinediones - SE

A

Fluid retention > peripheral oedema (risk is increased if insulin used concomitantly)

Increased risk of fractures

Increased risk of bladder Ca

114
Q

Diabetic Neuropathy

A

NICE updated their guidelines in 2013 and diabetic neuropathy is now managed in same way as neuropathic pain

1st line treatment = Amitryptilline, Duloxetine, Gabapentin, Pregabalin

If 1st line treatment doesn’t work, try one of other 3 drugs

  • Tramadol may be used as ‘rescue therapy’ for exacerbations of neuropathic pain
  • Topical Capsaicin may be used for localised neuropathic pain (e.g. post herpetic neuralgia)
  • Pain management clinics may be useful in patients with resistant problems

Gastroparesis:
Sx - erratic blood glucose control, bloating and vomiting
Mx: = Metoclopramide, Domperidone or Erythromycin (pro kinetics!)

115
Q

Diabetes Types

A

T1 = autoimmune mediated destruction of insulin producing beta cells of islets of langerhan

T2 = most common cause in developed world - relative deficiency of insulin due to excess adipose tissue > not enough insulin to go round excess fat > increased glucose

Pre-Diabetes = Patients who don’t yet meet criteria for formal diagnosis of T2DM but likely to develop condition in next few years. Require closer monitoring and lifestyle interventions e.g. weight loss

Gestational Diabetes = some pregnant women develop increased glucose during pregnancy. Important to detect as untreated may lead to adverse outcomes for mum and baby

MODY = Maturity onset diabetes of the young: inherited genetic disorders! - Affect production of insulin meaning younger patients present with Sx of T2DM i.e. asymptomatic hyperglycaemia with progression to more severe Cx e.g. DKA

LADA = Latent autoimmune diabetes of Adults - small group of patients who develop auto-immune related DM later in life. Often misdiagnosed as T2

Other Types = any pathological process which damages insulin producing cells of pancreas e.g. Chronic pancreatitis, Haemochromatosis, Excess glucocorticoid use

116
Q

T1DM vs T2DM Presentation

A

T1DM:

  • Weight loss
  • Polydipsia
  • Polyuria
  • May present with DKA (abdo pain, vomiting, reduced consciousness)

T2DM:

  • often picked up routinely on blood tests
  • polydipsia
  • polyuria

NB: Polydipsia and polyuria = water being ‘dragged out’ of body due to osmotic effects of excess blood glucose being excreted in the urine (glycosuria)

117
Q

Diabetes - Mx

A

Principles:

  • drug therapy to normalise blood glucose levels
  • monitoring for and treating any Cx related to diabetes
  • modifying any other RF for other conditions such as CVD

T1DM:

  • Always require insulin (there is an absolute deficiency of insulin w no pancreatic tissue left to stimulate with drugs)
  • Diff types of insulin according to duration of action

T2DM:
- Controlled using oral meds
1st line = metformin
2nd line = sulfonylureas, gliptins, pioglitazone
If oral meds are not controlling blood glucose sufficiently then insulin is used

NB Mx of DM represents 8% of NHS budget

118
Q

Diabetic Nephropathy - Stages

A

5 Stages (for T1DM primarily, T2DM progress through similar stages but on a diff timescale)

Stage 1: Hyperfiltration: increase in GFR (elevation of GFR usually persists into Stage 2) - may be reversible!

Stage 2: (Silent or latent phase)
Most patients do not develop microalbuminuria for 10 years, GFR remains elevated

Stage 3: (Incipient nephropathy)
Microalbuminuria (albumin excretion of 30-300mg/d, dipstick negative)

Stage 4: (Overt Nephropathy)
Persistent proteinuria (albumin excretion >300mg/d, dipstick +ve)
HTN present in most patients
Histology - Diffuse glomerulosclerosis and focal glomerulosclerosis (Kimmelstiel Wilson nodules)

Stage 5: (End Stage Renal Disease)
GFR typically <10mL/min
Renal replacement therapy needed

119
Q

LADA - Latent Autoimmune Diabetes of Adulthood

A

= Disorder in which, despite the presence of islet antibodies at diagnosis of DM, the progression of autoimmune cell failure is SLOW

In contrast to T2DM, patients are typically younger and without an increased body habitus

In contrast to T1DM, insulin is not usually required in the early stages of the disease

Diagnosis:
- may be aided through glutamic acid decarboxylase (GAD) as well as autoantibodies tests and evidence of other autoimmune diseases

120
Q

HHS

A
HHS confirmed by:
- Dehydration
- Osmolality > 320mosmol/kg
- Hyperglycaemia >30mmol/L
with pH > 7.3, bicarb >15mmol/L and no significant ketonaemia <3mmol/L
121
Q

Meglitinides in T2DM

A
  • stimulate insulin release
  • useful for post-prandial hyperglycaemia
  • good for erratic lifestyle
  • patients take them early before meals

eg Repaglinide

122
Q

Diabetes Diagnosis

A

Fasting glucose >= 7.0mmol/L

Random glucose >= 11.1mmol/L (or after 75g OGTT)

If asymptomatic, NEED 2 READINGS

123
Q

Starting Metformin

A

Metformin should be titrated slowly

Eg Metformin 500mg OD w food for 14d then 500mg BD for 14d then review

Leave at least a week before increasing dose

GI SE more common if not titrated slowly

124
Q

Target Blood Sugars in T1DM on Insulin

A

NICE recommends that people with type 1 diabetes should aim for 5-7mmol/litre on waking, 4-7mmol/litre before meals and 5-9mmol/litre 90 minutes after eating.

125
Q

Blood Pressure Targets in Diabetics

A

For a patient with hypertension without any other comorbodities, a target of less than 140/90mmHg is recommended. If the patient also has diabetes then a target of less than 140/80mmHg is recommended, whilst if there is diabetes and organ damage (kidney, eye or cerebrovascular) then a target of less than 130/80mmHg is recommended.

126
Q

How to measure Osmolality?

A

Serum Osmolality = (2 x (Na + K)) + (BUN (Blood Urea Nitrogen) / 2.8) + (glucose / 18)

Osmolality can be calculated with the formula 2(Na+K) +glucose+urea

In a Question work it out with

2(Na+ + K+) + UREA + GLUCOSE = OSMOLALITY

IF >320 = Probably HHS

127
Q

Indication for Insulin in HHS

A

Fluid replacement must commence first; an initial insulin bolus of 0.15 U per kg may be given once infusions are underway. Fluid replacement alone with 0.9% sodium chloride solution will result in falling blood glucose. Insulin treatment prior to adequate fluid replacement may result in cardiovascular collapse as water moves out of the intravascular space, with a resulting decline in intravascular volume.

Patients with HHS are often exquisitely sensitive to insulin and require much lower doses than in diabetic ketoacidosis (DKA).The recommended insulin dose is a fixed rate intravenous insulin infusion (FRIII) given at 0.05 units per kg per hour (e.g. 4 units/hr in an 80 kg man) is used

128
Q

HbA1c Target Post Pancreatectomy

A

A more lax HbA1c target is instigated for patients who have a history of pancreatectomy, of 53.

129
Q

Resolution of DKA

A

According to the Joint British Diabetes Societies Inpatient Care Group, resolution of DKA is defined as:
pH >7.3
bicarbonate > 15 mmol/L
ketones < 0.6 mmol/L

Note that blood glucose is NOT part of the criteria in defining resolution of DKA. On resolution, insulin infusion should be stopped and the patient should be restarted on their previous insulin regime with input from the diabetic team to consider readjustment of doses. The underlying cause of DKA should also be sought. In newly diagnosed patients, subcutaneous insulin glargine (lantus) should be consider until further diabetic review.

130
Q

HIV, T2DM and Metformin

A

Patients with HIV are at increased risk of lactic acidosis and hence it is prudent to check lactate levels before starting metformin in such patients.

Lactic acidaemia with no or mild symptoms was detected in 8% to 21% of patients receiving at least 1 nucleoside reverse transcriptase inhibitor, versus 0% to 1% of patients receiving no antiretroviral therapy. Symptomatic lactic acidaemia is less common (occurring in about 1.5%-2.5%).

Monitoring for lactic acidosis is recommended in the initial few months of metformin therapy. Patients with impaired renal function(serum creatinine>1.5 mg/dl OR creatine >132.6umol/L) or with a venous lactate level more than twice the normal level prior to treatment should not be started on metformin.

131
Q

Choice of Anti-Hypertensive in Afro-Caribbean Diabetic

A

Afro-Caribbeans and Diabetes with HTN

The diabetes makes an ACE inhibitor a better antihypertensive choice, whilst the Afro-Caribbean ethnicity means that ACE inhibitors are less likely to be effective. In this situation, NICE guidelines suggest to start a combination of an ACE inhibitor and a calcium-channel blocker or a diuretic. Afro-Caribbean ethnicity is associated with a genetic polymorphism of ACE making ACE inhibitors less likely to be effective for controlloing blood pressure.

132
Q

At what rate should you aim to lower blood glucose in DKA?

A

The most recent guidelines by the Joint British Diabetes Societies Inpatient Care Group in September 2012 recommends reduction in blood glucose of 3 mmol/l per hour, until BM reaches 14 mmol/l, at which point 5% dextrose should be considered as the intravenous fluid of choice. Rapid glucose lowering should be avoided: the rapid flux in osmolality can result in significant cerebral oedema and resultant cerebral damage.

133
Q

Insulin Therapy - Overnight Hypos and Low Morning Fasting Glucose

A

Altering e.g. a split of insulin to the conventional 2/3rd in the morning and 1/3rd in the evening reduces the risk of overnight hypos.

134
Q

Conditions where HbA1c cannot be used for diagnosis of T2DM

A
Conditions where HbA1c may not be used for diagnosis: 
haemoglobinopathies 
haemolytic anaemia 
untreated iron deficiency anaemia 
suspected gestational diabetes 
children 
HIV 
chronic kidney disease
135
Q

HHS - Mx and Risk of Cerebral Oedema

A

Hyperosmolar hyperglycaemic coma.

The current guidance advises treatment initially with normal saline intravenous rehydration. The target reduction in osmolality is 3-8 mosmol/kg an hour. Only if this target is not being met and the glucose level not reducing sufficiently is insulin to be started.

136
Q

Why potassium is so important in DKA

A

Serum potassium derangements are common and need careful management. Potassium is driven into cells by insulin. Serum potassium levels are therefore often high on presentation while blood insulin levels are depleted. Despite this, total body potassium is low due to fluid losses and requires careful monitoring and replacement during treatment.

137
Q

Diseases that can result in Diabetes

A

NB Any pathological process which damages insulin producing cells of pancreas e.g. Chronic pancreatitis, Haemochromatosis, Excess glucocorticoid use

138
Q

DVLA and Diabetes

A

DVLA: diabetes mellitus

Until recently people with diabetes who used insulin could not hold a HGV licence. The DVLA changed the rules in October 2011. The following standards need to be met (and also apply to patients using other hypoglycaemic inducing drugs such as sulfonylureas):
there has not been any severe hypoglycaemic event in the previous 12 months
the driver has full hypoglycaemic awareness
the driver must show adequate control of the condition by regular blood glucose monitoring*, at least twice daily and at times relevant to driving
the driver must demonstrate an understanding of the risks of hypoglycaemia
here are no other debarring complications of diabetes

From a practical point of view patients on insulin who want to apply for a Group 2 (HGV) licence need to complete a VDIAB1I form.

Other specific points for group 1 drivers:
if on insulin then patient can drive a car as long as they have hypoglycaemic awareness, not more than one episode of hypoglycaemia requiring the assistance of another person within the preceding 12 months and no relevant visual impairment. Drivers are normally contacted by DVLA
if on tablets or exenatide no need to notify DVLA. If tablets may induce hypoglycaemia (e.g. sulfonylureas) then there must not have been more than one episode of hypoglycaemia requiring the assistance of another person within the preceding 12 months
if diet controlled alone then no requirement to inform DVLA

*to demonstrate adequate control, the Secretary of State’s Honorary Medical Advisory Panel on Diabetes Mellitus has recommended that applicants will need to have used blood glucose meters with a memory function to measure and record blood glucose levels for at least 3 months prior to submitting their application. The patient must then be reviewed annually by a diabetes consultant, with 3 months of blood glucose monitoring data available. A 1 year license is then issued annually.

139
Q

DVLA and Diabetes - Example Question

A

A 53 year old obese HGV driver, normally taking BD novomix 30 insulin presents to your outpatient clinic to clarify some driving regulations he had overheard while eating with colleageus. He is extremely tearful and anxious. He is worried about losing his livelihood as a result of his diabetes.

He was first diagnosed with type 2 diabetes 9 years ago and became insulin dependent 2 years ago. He reports good compliance with insulin every day. However, 18 months ago, he took the same units of insulin after exercising and felt giddy. A spot blood glucose check demonstrated 2.8 mmol/l, which improved immediately after drinking lucozade that he carried with him. No hospitalisation was required. He has no other past medical history. He has no visual field or peripheral nerve impairments. What is the advice you give him regarding driving?

> Can continue driving, review in 1 year
Can continue driving, no further reviews required
Must stop driving and give up license permanently.
Must stop driving temporarily and review in 6 months
Patient can drive type 1 vehicles (cars, motorcycles) but not type 2 vehicles (lorries, HGV) and should reconsider his profession

The salient points in this case history are that the patient, although insulin dependent for treating his type 2 diabetes, retains hypoglycaemia awareness. A recent change in the DVLA guidelines of May 2012 allows HGV drivers to retain their license even if taking insulin, provided they have not suffered from hypoglycaemia requiring the assistance of others within the past 12 months, and the patient has no visual field impairments. The patient must then be reviewed annually by a diabetes consultant, with 3 months of blood glucose monitoring data available. A 1 year license is then issued annually.

140
Q

T2DM and Starting Insulin

A

According to NICE guidelines which type of insulin should be tried initially in patients with T2DM starting insulin for the first time?

= Human NPH Insulin (Isoprene, Intermediate acting) taken at bedtime or BD according to need

141
Q

How to choose which anti-diabetic drug to add to Metformin?

A

GLIPTIN: (eg Sitagliptin, DPP-4 inhibitor)

  • best option for drivers and obese
  • No risk of hypoglycaemia
  • weight neutral

SULFONYLUREA (eg Gliclazide)

  • Risk of hypoglycaemia
  • Weight gain

GLITAZONE (Eg Pioglitazone, Thiazolidinedione)
- Weight gain
- Fluid retention
CI in heart failure

SGLT-2 INHIBITOR (Eg Dapagliflozin)
- Glycosuria > UTIs and Dehydration

If triple therapy not effective and BMI >35
> Metformin + X + GLP-1 mimetic (Exenatide)

NB Risk of pancreatitis with Exenatide

142
Q

SGLT2 inhibitors and HF

A

Evidence for empagliflozin eg one of the SGLT2 inhibitor class, is that it promotes sodium excretion and has a thiazide diuretic like effect with some benefit in patients with incipient cardiac failure.

143
Q

DKA T2DM - Example Question

A

A 72-year-old female presents with 5 days of general decline following a recent urinary tract infection, treated with oral antibiotics in the community by the GP. She is known to be a type 2 diabetic, diagnosed 28 years ago and insulin dependent for the past 6 years. She is normally on 46 units Lantus, 23 units TDS Novorapid. On examination, she is not orientated in time or place, GCS 14/15. She has no focal neurology, chest and cardiovascular auscultation are unremarkable. You demonstrate suprapubic tenderness on deep palpation but the abdomen is other soft and non-tender, bowel sounds are present. She appears extremely dehydrated: her mucous membranes are dry, peripheries cool with capillary refill time of 4 seconds and JVP +1 cm above the angle of Louis. Her blood sugar is 31mmol/L and a venous blood gas demonstrates pH 7.22, lactate 2 mmol/l, ketones 5 mmol/l. A urine dip is awaited. What is the most likely diagnosis?

Hyperglycaemic hyperketotic state
Hyperglycaemia secondary to poor medical compliance during recent acute illness
Urosepsis secondary to inadequately treated UTI
> Diabetic ketoacidosis
Dehydration secondary to poor oral intake

The patient is acidotic with ketones >3 mmol/l demonstrated, on a background of known insulin dependence. Although she is known to be a type 2 diabetic, it should be remembered that both types of diabetics can present as DKA, particularly advanced T2 DM who produce little to no endogenous insulin and are hence unable to shut down ketogenesis. Treatment should be as per DKA protocols, with intravenous fluids, fixed rate insulin infusion @ 0.1 unit/kg/hour, thromboprophylaxis, broad spectrum antibiotics and appropriate K+ replacement with insulin1.